Расчет электрической мощности по току и напряжению: Расчет мощности по току и напряжению

Содержание

Формула расчета мощности электрического тока

При проектировании любых электрических цепей выполняется расчет мощности. На его основе производится выбор основных элементов и вычисляется допустимая нагрузка. Если расчет для цепи постоянного тока не представляет сложности (в соответствии с законом Ома, необходимо умножить силу тока на напряжение – Р=U*I), то с вычислением мощности переменного тока – не все так просто. Для объяснения потребуется обратиться к основам электротехники, не вдаваясь в подробности, приведем краткое изложение основных тезисов.

Полная мощность и ее составляющие

В цепях переменного тока расчет мощности ведется с учетом законов синусоидальных изменений напряжения и тока. В связи с этим введено понятие полной мощности (S), которая включает в себя две составляющие: реактивную (Q) и активную (P). Графическое описание этих величин можно сделать через треугольник мощностей (см. рис.1).

Под активной составляющей (Р) подразумевается мощность полезной нагрузки (безвозвратное преобразование электроэнергии в тепло, свет и т.д.). Измеряется данная величина в ваттах (Вт), на бытовом уровне принято вести расчет в киловаттах (кВт), в производственной сфере – мегаваттах (мВт).

Реактивная составляющая (Q) описывает емкостную и индуктивную электронагрузку в цепи переменного тока, единица измерения этой величины Вар.

Рис. 1. Треугольник мощностей (А) и напряжений (В)

В соответствии с графическим представлением, соотношения в треугольнике мощностей можно описать с применением элементарных тригонометрических тождеств, что дает возможность использовать следующие формулы:

  • S = √ P 2 +Q 2 , – для полной мощности;
  • и Q = U*I*cos⁡ φ , и P = U*I*sin φ – для реактивной и активной составляющих.

Эти расчеты применимы для однофазной сети (например, бытовой 220 В), для вычисления мощности трехфазной сети (380 В) в формулы необходимо добавить множитель – √ 3 (при симметричной нагрузке) или суммировать мощности всех фаз (если нагрузка несимметрична).

Для лучшего понимания процесса воздействия составляющих полной мощности давайте рассмотрим «чистое» проявление нагрузки в активном, индуктивном и емкостном виде.

Активная нагрузка

Возьмем гипотетическую схему, в которой используется «чистое» активное сопротивление и соответствующий источник переменного напряжения. Графическое описание работы такой цепи продемонстрировано на рисунке 2, где отображаются основные параметры для определенного временного диапазона (t).

Емкостная нагрузка

Как видно на рисунке 3, график характеристик емкостной нагрузки несколько отличается от активной.

Индуктивная нагрузка

Представленный ниже график демонстрирует характер «чистой» индуктивной нагрузки. Как видим, изменилось только направление мощности, что касается наращения, оно равно нулю.

Негативное воздействие реактивной нагрузки

В приведенных выше примерах рассматривались варианты, где присутствует «чистая» реактивная нагрузка. Фактор воздействия активного сопротивления в расчет не принимался. В таких условиях реактивное воздействие равно нулю, а значит, можно не принимать его во внимание. Как вы понимаете, в реальных условиях такое невозможно. Даже, если гипотетически такая нагрузка бы существовала, нельзя исключать сопротивление медных или алюминиевых жил кабеля, необходимого для ее подключения к источнику питания.

Реактивная составляющая может проявляться в виде нагрева активных компонентов цепи, например, двигателя, трансформатора, соединительных проводов, питающего кабеля и т.д. На это тратится определенное количество энергии, что приводит к снижению основных характеристик.

Реактивная мощность воздействует на цепь следующим образом:

  • не производит ни какой полезной работы;
  • вызывает серьезные потери и нештатные нагрузки на электроприборы;
  • может спровоцировать возникновение серьезной аварии.

Именно по этому, производя соответствующие вычисления для электроцепи, нельзя исключать фактор влияния индуктивной и емкостной нагрузки и, если необходимо, предусматривать использование технических систем для ее компенсации.

Расчет потребляемой мощности

В быту часто приходится сталкиваться с вычислением потребляемой мощности, например, для проверки допустимой нагрузки на проводку перед подключением ресурсоемкого электропотребителя (кондиционера, бойлера, электрической плиты и т.д.). Также в таком расчете есть необходимость при выборе защитных автоматов для распределительного щита, через который выполняется подключение квартиры к электроснабжению.

В таких случаях расчет мощности по току и напряжению делать не обязательно, достаточно просуммировать потребляемую энергию всех приборов, которые могут быть включены одновременно. Не связываясь с расчетами, узнать эту величину для каждого устройства можно тремя способами:

  1. обратившись к технической документации устройства;
  2. посмотрев это значение на наклейке задней панели; Потребляемая мощность прибора часто указывается на тыльной стороне
  3. воспользовавшись таблицей, где указано среднее значение потребляемой мощности для бытовых приборов.

Таблица значений средней потребляемой мощности

При расчетах следует учитывать, что пусковая мощность некоторых электроприборов может существенно отличаться от номинальной. Для бытовых устройств этот параметр практически никогда не указывается в технической документации, поэтому необходимо обратиться к соответствующей таблице, где содержатся средние значения параметров стартовой мощности для различных приборов (желательно выбирать максимальную величину).

Пожаловалась бабушка соседка снизу: подарили мне дети моющий пылесос. Он прекрасно работает, но откуда-то идет запах гари.

Пошел смотреть. Проводка у нас старая: лапша из алюминия 2,5 квадрата. А пылесос потребляет 2,5 kW. Прикинул, как работает формула расчета мощности по току и напряжению для этого случая.

Разделил 2500 ватт на 220 вольт. Получил чуть больше 11 ампер. Наши провода держат нагрузку 22 А. Имеем практически двойной резерв потоку. Другие потребители при уборке отключены.

Стали проверять и нюхать: запах около квартирного щитка. Открыл, осмотрел: шина сборки ноля в саже, на одной перемычке горелая изоляция. Винт крепления ослаблен. Вот и причина начала возгорания. Исправил.

На этом примере я показываю, что всегда надо оценивать мощность потребления электроприборов и возможности проводки с защитными устройствами. Об этом рассказываю ниже.

Что такое мощность в электричестве: просто о сложном

Вспомнилась былина об Илье Муромце, когда он приложил всю свою мощь к соловью разбойнику. У бедолаги сразу посыпались искры из глаз, как пламя с верхней картинки на проводке с неправильным монтажом.

Простыми словами: мощность в электричестве — это силовая характеристика энергии, которой оценивают, как способности генераторных установок ее вырабатывать, так возможности потребителей и транспортных магистралей.

Все эти участки должны быть точно смонтированы и налажены для обеспечения безопасной работы. Как только в любом месте возникает неисправность, так сразу развивается авария во всей схеме.

Если говорить о домашнем электрическом оборудовании, то приходится постоянно соблюдать баланс между:

  1. включенными в сеть приборами;
  2. конструкцией проводов и кабелей;
  3. настройкой защитных устройств.

Только комплексное решение этих трех вопросов может обеспечить безопасность проводки и жильцов.

Как рассчитать электрическую мощность в быту

Формулы расчета мощности в электричестве позволяют выполнить качественную оценку безопасности каждого из перечисленных выше пунктов.

Пользоваться ими не сложно. Я уже приводил в предыдущих статьях шпаргалку электрика, где они помещены в наглядной форме для цепей постоянного тока.

Они полностью справедливы для активной составляющей мощности переменного тока, совершающей полезную работу. Кстати, кроме нее есть еще и бесполезная — реактивная, связанная с потерями энергии. Ее описанию посвящен второй раздел.

Такие вычисления удобно делать с помощью онлайн калькулятора. Он избавляет от рутинных математических вычислений и арифметических ошибок.

При любом из способов для расчета активной мощности требуется знать две из трех электрических величин:

Как измерить электрическую мощность дома

Существует еще одна возможность оценки активной мощности: ее измерение в действующей схеме специальными приборами: ваттметрами.

Точные замеры может обеспечить промышленный лабораторный ваттметер. Он изготавливается как прибор, работающий на аналоговых сигналах,так и с помощью цифровых технологий.

В бытовой проводке точные вычисления не нужны. Для нее выпускаются различные виды более простых ваттметров.

Популярностью пользуются приборы, которые можно вставить в розетку и подключить к ним шнур питания от потребителя, включить их в работу и сразу снять показания на дисплее в ваттах.

Их так и называют: ваттметр розетка. Они измеряют чисто активную мощность переменного тока.

Такие приборы избавляют электрика от выполнения сложных операций под напряжением, когда требуется замерять:

  • действующее напряжение;
  • силу тока;
  • угол сдвига фаз между векторами тока и напряжения.

Потом все данные дополнительно требуется вводить в формулу расчета мощности по току и напряжению, делать по ней вычисления.

Этот метод можно упростить, если внимательно наблюдать за показаниями электрического счетчика индукционной системы с вращающимся диском. Он считает совершенную работу: потребленную мощность за определенную время.

Однако скорость вращения диска как раз и характеризует величину потребления. Надо просто посчитать сколько раз он обернется за минуту и перевести в ватты по табличке, расположенной на корпусе.

Почему реактивное сопротивление схемы влияет на мощность переменного тока

Синусоидальная гармоника напряжения, поступая на резистивное сопротивление, изменяет величину тока без его отклонения на комплексной плоскости.

Такой ток совершает полезную работу с минимальными потерями энергии, вырабатывая активную мощность. Частота колебания сигнала не оказывает на нее никакого влияния.

Сопротивление конденсатора и индуктивности зависит от частоты гармоники. Его противодействие отклоняет направление тока на каждом из этих элементов в разные стороны.

Такие процессы связаны с потерей части энергии на бесполезные преобразования. На них расходуется мощность Q, которую называют реактивной.Ее влияние на полную мощность S и связь с активной P удобно представлять графически прямоугольным треугольником.

Захотелось его нарисовать на фоне оборудования из нагромождений фарфора и металла, где пришлось поработать довольно долго.Отвлекся. Не судите за это строго.

Сравните его с опубликованным мною ранее треугольником сопротивлений. Находите общие черты?

Ими являются геометрические пропорции фигуры, описывающие их формулы и угол φ, определяющий потери полной мощности. Перехожу к их более подробному рассмотрению.

Формулы расчета мощности для однофазной и трехфазной схемы питания

В идеальном теоретическом случае трехфазная схема состоит из трех одинаковых однофазных цепей. На практике всегда есть какие-то отклонения. Но, в большинстве случаев при анализах ими пренебрегают.

Поэтому рассматриваем вначале наиболее простой вопрос.

Графики и формулы под однофазное напряжение

Как работает резистор

На чисто резистивном сопротивлении синусоиды тока и напряжения совпадают по углу, направлены на каждом полупериоде одинаково.Поэтому их произведение, выражающее мощность, всегда положительно.

Его значение в произвольный момент времени t называют мгновенным, обозначая строчной буквой p.

Среднее значение мощности в течение одного периода называют активной составляющей. Ее график для переменного тока имеет фигуру симметричного всплеска с максимальным значением Pm в середине каждого полупериода Т/2.

Если взять половину его величины Pm/2 и провести прямую линию в течении одного периода Т, то получим прямоугольник с ординатой P.

Его площадь равна двум площадям графиков активной составляющих одного любого полупериода. Если посмотреть на картинку внимательнее, то можно представить, что верхняя часть всплеска отрезана,перевернута и заполнила свободное пространство внизу.

Представление этого графика помогает запомнить, что на активном сопротивлении мощность постоянного и переменного тока вычисляется по одной формуле, не меняет своего знака.

На резисторе не создается реактивных потерь.

Как работает индуктивность

Катушка с обмоткой своими витками запасает энергию магнитного поля. Благодаря процессу ее накопления индуктивное сопротивление отодвигает вперед на 90 градусов вектор тока относительно приложенного напряжения на комплексной плоскости.

Перемножая их мгновенные величины получаем значения мощности, которое за один период меняет знаки (направление) в каждом полупериоде.

Частота изменения мощности на индуктивности в два раза выше,чем у ее составляющих: синусоид тока и напряжения. Она состоит из двух частей:

  1. активной, обозначаемой индексом PL;
  2. реактивной QL.

Реактивная часть на индуктивности создается за счет постоянного обмена энергией между катушкой и приложенным источником. На ее величину влияет значение индуктивного сопротивления XL.

Как работает конденсатор

Емкость конденсатора постоянно накапливает заряд между своими обкладками. За счет этого происходит сдвиг вектора тока вперед на 90 градусов относительно приложенного напряжения.

График мгновенной мощности напоминает вид предыдущего, но начинается с отрицательной полуволны.

Реактивная составляющая, выделяемая на конденсаторе, зависит от величины емкостного сопротивления XC.

Как работает реальная схема со всеми видами сопротивлений

В чистом виде приведенные выше графики и выражения встречаются не так часто. На самом деле передача электроэнергии и ее работа на переменном токе связаны с комплексным преодолением сил электрического сопротивления резисторов, конденсаторов и индуктивностей.

Причем, какая-то из этих составляющих будет преобладать. Для таких случаев преобразования электрической энергии в мгновенную мощность могут иметь один из следующих видов.

На верхней картинке показан случай, когда вектор тока отстает от приложенного напряжения, а на нижней — опережает.

В обоих случаях величина активной составляющей уменьшается от значения полной на значение, выражаемое как cosφ. Поэтому его принято называть коэффициентом мощности.

Как работает схема трехфазного электроснабжения

На ввод распределительного щита многоэтажного здания поступает трехфазное напряжение от электроснабжающей организации, вырабатываемое промышленными генераторами.

Его же, за отдельную плату, при желании может подключить владелец частного дома, что многие и делают. При этом рабочая схема и диаграмма напряжений выглядит следующим образом.

В старой системе заземления TN-C она выполняется четырехпроводным подключением, а у новой TN-S — пятипроводным с добавлением защитного РЕ проводника. Его на этой схеме я не показываю для упрощения.

Каждую из фаз при работе необходимо стараться нагружать одинаково равными по величине токами. Тогда в домашней проводке будет создаваться наиболее благоприятный оптимальный режим без опасных перекосов энергии.

В этом случае формула расчета мощности по току и напряжению для трехфазной схемы может быть представлена простой суммой аналогичных формул для составляющих однофазных цепей.

А поскольку они все идентичные, то их просто утраивают.

Например, когда активная мощность фазы В имеет выражением Рв=Uв×Iв×cosφ, то для всей трехфазной схемы она будет выражена следующей формулой:

Если пометить фазное выражение буквой ф. например Pф, томожно записать:

Аналогично будет вычисляться реактивная составляющая

Поскольку P и Q представляют величины катетов прямоугольного треугольника, то гипотенузу или полную составляющую можно вычислить как квадратный корень из суммы их квадратов.

Как учитывается трехфазная полная мощность

В энергосистеме, да и в частном доме, требуется анализировать подключенные нагрузки, равномерно распределять их по источникам напряжений.

С этой целью работают многочисленные конструкции измерительных приборов. На щитах управления подстанций расположены щитовые ваттметры и варметры, предназначенные для работы в разных долях кратности.

Старые аналоговые приборы показаны на этой картинке.

Для того, чтобы не путаться в записях вычислений введены разные наименования единиц. Они обозначаются:

  • ВА — (русское), VA (международное) вольтампер для полной величины мощности;
  • Вт —(русское), var (международное) ватт —активной;
  • вар (русское), var (международное) — реактивной.

Аналоговые приборы измеряют только активную или реактивную составляющую, а полную величину необходимо вычислять по формулам.

Многие современные цифровые приборы способны осуществлять эту функцию автоматически.

Видеоурок Павла Виктор дополняет мой материал. Рекомендую посмотреть.

Калькулятор мощности для своих

Здесь вы можете выполнить вычисления онлайн без использования формул и арифметических действий. Просто введите ваши исходные данные в таблицу и жмите кнопку “Рассчитать ток”.

А в заключение напоминаю, что для ваших вопросов создан раздел комментариев. Задавайте их, я отвечу.

Иногда можно услышать такой простой вопрос: «какая мощность в розетке?». Ответ, как ни странно, чаще всего такой: 10 ампер. Или – 220 вольт. Понятно, что вопрос – дурацкий. Но и объяснение не лучше – «А на розетке так написано».

Мощность и ток

Если правильно отвечать на поставленный вопрос, то для читателей, прогуливающих в детстве уроки физики, можно сказать, что мощность электричества зависит от двух величин:

  • величины напряжения;
  • силы тока.

В общем, эти две величины определяют величину мощности как переменного, так и постоянного тока. Память может подсказать что-то типа: для участка цепи, для полной цепи. Это отголоски того же школьного учебника физики, где говорится о законе Ома.

Да, этот знаменитый закон и позволяет рассчитать мощность электрического тока. Конечно, школьная программа представляла этот закон для цепей постоянного тока, но суть от этого не меняется. Формула вечная и неизменная: P = U х I.

Перефразируя закон ома в простой язык, получаем простой ответ на вопрос о мощности в розетке: сила тока зависит от нагрузки.

Сила тока и приложенная нагрузка

Тривиальное понятие этого тезиса позволит не производить элементарных действий, постоянно совершаемых нами, или окружающими нас людьми:

  • включать один электрический удлинитель в другой, втыкая в оба все доступные вилки от разных, иногда достаточно мощных, потребителей электроэнергии;
  • подключать к севшему аккумулятору автомобиля другой, соединяя их проводами от старой электропроводки;
  • наращивать провода от электрического чайника кабелем с витой парой;
  • устанавливать в гараже нагреватель, мощностью 5 квт, подключая его к обыкновенной розетке.

Аналогичные примеры неграмотных действий можно приводить до бесконечности. Человеческая беспечность не знает границ. Чтобы больше не допускать подобных ошибок, давайте разберем как правильно производить расчет электрической мощности.

Чайник и электрическая мощность

Не забивая головы простейшими формулами (есть дела и поважнее этого), запомним простое соотношение, достаточное для применения его в быту. Точность его не соответствует формуле расчета, но позволяет помнить, что: 1 квт электроэнергии – это приблизительно 5 ампер тока в сети 220 вольт.

Таким образом, становится понятно, что электрический чайник, включенный в кухонную розетку, потребляет около 5 ампер тока. А лампа накаливания, мощностью 100 Вт – в десять раз меньше: 0,5 ампера. Конечно, такие примитивные знания нужны для домохозяек, расчет мощности электрического тока производится по формулам.

Необходимость расчетов мощности

Человек мало сталкивается с необходимостью проведения расчетов (мощностей постоянного электрического тока) в быту. Чаще всего такая необходимость возникает при ремонте автомобиля, где источником тока служит аккумулятор. Или какой-то продвинутый пользователь начинает подбирать новый кулер для своего процессора в компьютере.

Чаще возникает необходимость провести элементарные расчеты при ремонтных работах в квартире, при подборе сгоревшего блока питания и пр.

Расчет мощности электрического тока по формулам

Существует формула расчета электрического тока для однофазной и трехфазной сети. Вряд ли кто-то захочет и сможет ими воспользоваться – разбираться что такое cosφ при замене электрической проводки в доме или квартире нецелесообразно.

Реально можно произвести все необходимые расчеты в режиме онлайн. Интернет набит разными таблицами, соответствующими графиками и калькуляторами. Для очень нуждающихся читателей можно добавить, что сечение кабеля для осветительной сети — 1,5 кв. мм. А для электропитания розеток применяется кабель сечением 2,5 кв. мм.

Остальные расчеты, требующиеся при производстве электромонтажных работ в различных областях деятельности – лучше доверить специалистам, которые в своей работе используют различные приборы: амперметры, вольтметры, индикаторы фазы, измерители сопротивления изоляции, измерители сопротивления заземления и пр.

Ремонт и строительство домов и квартир, особенности расчетов

Чтобы произвести расчет электропроводки в квартире недостаточно произвести подбор сечения электрических проводов. В электрическом щите устанавливаются и электрические автоматы, и защитные устройства и электрический счетчик. Эти установочные изделия также подбираются и рассчитываются при разработке проекта электропитания, в котором производится также расчет количества и параметров устройств защитного заземления.

Для расчетов и подбора видов электропроводки, использующейся при изготовлении удлинителей, организации временных схем электропитания, необходимо понимать, что силовые кабели для однофазной и трехфазной цепи различны по количеству жил, условиям прокладки, токовым нагрузкам и прочим параметрам.

При использовании кабелей и проводов необходимо учитывать и материал изготовления токопроводящих жил.

Наличие в загородном доме, даче трехфазных потребителей электроэнергии, таких как скважинный насос, электродвигатели, сварочное оборудование, требует при подборе кабелей электропроводки учитывать их пусковые токи. А при выборе электрического счетчика электроэнергии – активную и реактивную составляющую в потребляемой мощности, если предполагается постоянная работа трехфазного оборудования.

“>

Таблица расчета мощности по напряжению

При проектировании любых электрических цепей выполняется расчет мощности. На его основе производится выбор основных элементов и вычисляется допустимая нагрузка. Если расчет для цепи постоянного тока не представляет сложности (в соответствии с законом Ома, необходимо умножить силу тока на напряжение — Р=U*I), то с вычислением мощности переменного тока — не все так просто. Для объяснения потребуется обратиться к основам электротехники, не вдаваясь в подробности, приведем краткое изложение основных тезисов.

Полная мощность и ее составляющие

В цепях переменного тока расчет мощности ведется с учетом законов синусоидальных изменений напряжения и тока. В связи с этим введено понятие полной мощности (S), которая включает в себя две составляющие: реактивную (Q) и активную (P). Графическое описание этих величин можно сделать через треугольник мощностей (см. рис.1).

Под активной составляющей (Р) подразумевается мощность полезной нагрузки (безвозвратное преобразование электроэнергии в тепло, свет и т.д.). Измеряется данная величина в ваттах (Вт), на бытовом уровне принято вести расчет в киловаттах (кВт), в производственной сфере – мегаваттах (мВт).

Реактивная составляющая (Q) описывает емкостную и индуктивную электронагрузку в цепи переменного тока, единица измерения этой величины Вар.

Рис. 1. Треугольник мощностей (А) и напряжений (В)

В соответствии с графическим представлением, соотношения в треугольнике мощностей можно описать с применением элементарных тригонометрических тождеств, что дает возможность использовать следующие формулы:

  • S = √ P 2 +Q 2 , — для полной мощности;
  • и Q = U*I*cos⁡ φ , и P = U*I*sin φ — для реактивной и активной составляющих.

Эти расчеты применимы для однофазной сети (например, бытовой 220 В), для вычисления мощности трехфазной сети (380 В) в формулы необходимо добавить множитель – √ 3 (при симметричной нагрузке) или суммировать мощности всех фаз (если нагрузка несимметрична).

Для лучшего понимания процесса воздействия составляющих полной мощности давайте рассмотрим «чистое» проявление нагрузки в активном, индуктивном и емкостном виде.

Активная нагрузка

Возьмем гипотетическую схему, в которой используется «чистое» активное сопротивление и соответствующий источник переменного напряжения. Графическое описание работы такой цепи продемонстрировано на рисунке 2, где отображаются основные параметры для определенного временного диапазона (t).

Емкостная нагрузка

Как видно на рисунке 3, график характеристик емкостной нагрузки несколько отличается от активной.

Индуктивная нагрузка

Представленный ниже график демонстрирует характер «чистой» индуктивной нагрузки. Как видим, изменилось только направление мощности, что касается наращения, оно равно нулю.

Негативное воздействие реактивной нагрузки

В приведенных выше примерах рассматривались варианты, где присутствует «чистая» реактивная нагрузка. Фактор воздействия активного сопротивления в расчет не принимался. В таких условиях реактивное воздействие равно нулю, а значит, можно не принимать его во внимание. Как вы понимаете, в реальных условиях такое невозможно. Даже, если гипотетически такая нагрузка бы существовала, нельзя исключать сопротивление медных или алюминиевых жил кабеля, необходимого для ее подключения к источнику питания.

Реактивная составляющая может проявляться в виде нагрева активных компонентов цепи, например, двигателя, трансформатора, соединительных проводов, питающего кабеля и т.д. На это тратится определенное количество энергии, что приводит к снижению основных характеристик.

Реактивная мощность воздействует на цепь следующим образом:

  • не производит ни какой полезной работы;
  • вызывает серьезные потери и нештатные нагрузки на электроприборы;
  • может спровоцировать возникновение серьезной аварии.

Именно по этому, производя соответствующие вычисления для электроцепи, нельзя исключать фактор влияния индуктивной и емкостной нагрузки и, если необходимо, предусматривать использование технических систем для ее компенсации.

Расчет потребляемой мощности

В быту часто приходится сталкиваться с вычислением потребляемой мощности, например, для проверки допустимой нагрузки на проводку перед подключением ресурсоемкого электропотребителя (кондиционера, бойлера, электрической плиты и т.д.). Также в таком расчете есть необходимость при выборе защитных автоматов для распределительного щита, через который выполняется подключение квартиры к электроснабжению.

В таких случаях расчет мощности по току и напряжению делать не обязательно, достаточно просуммировать потребляемую энергию всех приборов, которые могут быть включены одновременно. Не связываясь с расчетами, узнать эту величину для каждого устройства можно тремя способами:

  1. обратившись к технической документации устройства;
  2. посмотрев это значение на наклейке задней панели; Потребляемая мощность прибора часто указывается на тыльной стороне
  3. воспользовавшись таблицей, где указано среднее значение потребляемой мощности для бытовых приборов.

Таблица значений средней потребляемой мощности

При расчетах следует учитывать, что пусковая мощность некоторых электроприборов может существенно отличаться от номинальной. Для бытовых устройств этот параметр практически никогда не указывается в технической документации, поэтому необходимо обратиться к соответствующей таблице, где содержатся средние значения параметров стартовой мощности для различных приборов (желательно выбирать максимальную величину).

Чтобы электропроводка и все электрическое оборудование, которое имеется в доме, работало исправно и правильно, необходимо правильно сделать вычисление мощности по току и электронапряжению, поскольку при неправильно подобранных показателях может возникнуть короткое замыкание или возгорание. Как сделать расчёт потребляемой мощности по току и напряжению, как вычисляется сила тока, формула через мощность и напряжение и другое, далее.

Как узнать силу тока, зная мощность и напряжения

Чтобы ответить на вопрос, как определить ток, необходимо поделить электронапряжение на общее число ватт. При этом сделать все необходимые вычисления можно самостоятельно, а можно прибегнуть к специальному онлайн-калькулятору.

Узнать потребление электроэнергии по токовой силе резистора можно умножением первой на сопротивление, выражаемое в Омах. В итоге, получится значение, представленное в вольтах, перемноженных на ом. Получится ампер.

Обратите внимание! Если нет сопротивления, нужно поделить ваттный показатель на токовую энергию, то есть следует поделить ватты на амперы и получится значение электроэнергии в вольтах. Понять мощностное показание через величину электричества с электронапряжением, можно умножив соответствующие показания с устройства.

Формулы для расчета тока в трехфазной сети

Подсчитать токовую энергию в трехфазной сети сложно, поскольку вместе одной фазы есть три. К тому же, сложность заключается в использовании нескольких схем соединения. Трудность состоит в симметрии или ее отсутствии во время распределения нагрузки по фазам.

Для определения силы тока в трехфазной сети, нужно общее число ватт поделить на показатель 1,73, перемноженный на напряжение и косинус мощностного коэффициента, который отражает активную и реактивную составляющую сопротивления нагрузки. Что касается однофазной сети, то из выражения для подсчета убирается показатель 1,73. Остается формула I = P/(U*cos φ).

Как рассчитать ампераж

Ампераж является значением электротока, которое выражена в амперах. Рассчитать ампераж можно так: I=P/U.

Расчет потребляемой мощности

Электромощность является величиной, которая отвечает за факт скорости изменения или передачи электрической энергии. Есть полная и активная мощностная нагрузка, а также активная и реактивная. Полная вычисляется так: S = √ (P2 + Q2), где P является активной частью, а Q реактивной. Для нахождения потребляемого мощностного показателя необходимо знать число электротока, которое потребляется нагрузкой, а также питательное напряжение, которое выдается при помощи источника.

Что касается бытового определения потребляемой электрической энергии, необходимо вычислить общее количество ватт питания электрических приборов и паспортные данные номинальной силы электротока котла. Как правило, все электрические приборы работают с переменным током и напряжением в 220 вольт. Для вычисления тока проще всего воспользоваться амперметром. Зная первый и второй параметры, реально узнать величину потребляемой энергии.

Стоит указать, что измерить мощность через напряжение или сделать расчет мощности по сопротивлению и напряжению возможно не только формулой, но и прибором. Для этого можно воспользоваться мультиметром с токоизмерительными клещами или специализированным измерителем — ваттметром.

Обратите внимание!

Оба работают по одному и тому же принципу, указанному в руководстве по их эксплуатации.

Мощность, ток и напряжение — три составляющие расчета проводки в доме. Узнать все необходимые параметры в любой сети просто при помощи формул, представленных выше. От этих значений будет зависеть исправность работы всей домашней электрики и безопасность ее владельца.

Чтобы обезопасить себя при работе с бытовыми электроприборами, необходимо в первую очередь правильно вычислить сечение кабеля и проводки. Потому-что если будет неправильно выбран кабель, это может привести к короткому замыканию, из за чего может произойти возгорание в здание, последствия могут быть катастрофическими.

Это правило относиться и к выбору кабеля для электродвигателей.

Расчёт мощности по току и напряжению

Данный расчет происходит по факту мощности, проделывать его необходимо еще до начала проектирование своего жилища (дома, квартиры).

  • Из этого значение зависят кабеля питающие приборы которые подключены к электросети.
  • По формуле можно вычислить силу тока, для этого понадобиться взять точное напряжение сети и нагрузку питающихся приборов. Ее величина дает нам понять площадь сечение жил.

Если вам известны все электроприборы, которые в будущем должны питаться от сети, тогда можно легко сделать расчеты для схемы электроснабжение. Эти же расчеты можно выполнять и для производственных целей.

Однофазная сеть напряжением 220 вольт

Формула силы тока I (A — амперы):

Где P — это электрическая полная нагрузка (ее обозначение обязательно указывается в техническом паспорте данного устройства), Вт — ватт;

U — напряжение электросети, В (вольт).

В таблице представлены стандартные нагрузки электроприборов и потребляемый ими ток (220 В).

ЭлектроприборПотребляемая мощность, ВтСила тока, А
Стиральная машина2000 – 25009,0 – 11,4
Джакузи2000 – 25009,0 – 11,4
Электроподогрев пола800 – 14003,6 – 6,4
Стационарная электрическая плита4500 – 850020,5 – 38,6
СВЧ печь900 – 13004,1 – 5,9
Посудомоечная машина2000 — 25009,0 – 11,4
Морозильники, холодильники140 — 3000,6 – 1,4
Мясорубка с электроприводом1100 — 12005,0 — 5,5
Электрочайник1850 – 20008,4 – 9,0
Электрическая кофеварка6з0 — 12003,0 – 5,5
Соковыжималка240 — 3601,1 – 1,6
Тостер640 — 11002,9 — 5,0
Миксер250 — 4001,1 – 1,8
Фен400 — 16001,8 – 7,3
Утюг900 — 17004,1 – 7,7
Пылесос680 — 14003,1 – 6,4
Вентилятор250 — 4001,0 – 1,8
Телевизор125 — 1800,6 – 0,8
Радиоаппаратура70 — 1000,3 – 0,5
Приборы освещения20 — 1000,1 – 0,4

На рисунке вы можете видет схему устройства электроснабжение дома при однофазном подключении к сети 220 вольт.

Схема приборов при однофазном напряжении

Как и показано на рисунке, все потребители должны быть подключены к соответствующим автоматам и счетчику, далее к общему автомату который будет выдерживать общею нагрузку дома. Кабель который будет доводит ток, должен выдерживать нагрузку всех подключенных бытовых приборов.

В таблице ниже показана скрытая проводка при однофазной схеме подключение жилища для подбора кабеля при напряжении 220 вольт.

Сечение жилы провода, мм 2Диаметр жилы проводника, ммМедные жилыАлюминиевые жилы
Ток, АМощность, ВтТок, АМощность, кВт
0,500,8061300
0,750,98102200
1,001,13143100
1,501,38153300102200
2,001,60194200143100
2,501,78214600163500
4,002,26275900214600
6,002,76347500265700
10,003,575011000388400
16,004,5180176005512100
25,005,64100220006514300

Как и показано в таблице, сечение жил зависит и от материала из которого изготовлен.

Трёхфазная сеть напряжением 380 В

В трехфазном электроснабжении сила тока рассчитывается по следующей формуле:

I =”” P /1,73 U

P — потребляемая мощность в ватах;

U — напряжение сети в вольтах.

В техфазной схеме элетропитания 380 В, формула имеет следующий вид:

I =”” P /657, 4

Если к дому будет проводиться трехфазная сеть 380 В, то схема подключения будет иметь следующий вид.

В таблице ниже представлена схема сечения жил в питающем кабеле при различной нагрузке при трехфазном напряжении 380 В для скрытой проводки.

Сечение жилы провода, мм 2Диаметр жилы проводника, ммМедные жилыАлюминиевые жилы
Ток, АМощность, ВтТок, АМощность, кВт
0,500,8062250
0,750,98103800
1,001,13145300
1,501,38155700103800
2,001,60197200145300
2,501,78217900166000
4,002,262710000217900
6,002,763412000269800
10,003,5750190003814000
16,004,5180300005520000
25,005,64100380006524000

Для дальнейшего расчета питания в цепях нагрузки, характеризующейся большой реактивной полной мощностью, что характерно применению электроснабжения в промышленности:

  • электродвигатели;
  • индукционные печи;
  • дроссели приборов освещения;
  • сварочные трансформаторы.

Это явление в обязательном порядке необходимо учитывать при дальнейших расчетах. В более мощных электроприборах нагрузка идет гораздо больше, поэтому в расчетах коэффициент мощности принимают 0,8.

При подсчете нагрузки на бытовые приборы запас мощности нужно брать 5%. Для электросети этот процент становит 20%.

>

Как найти мощность, зная силу тока, напряжение и сопротивление

В физике достаточно много внимания уделено энергии и мощности устройств, веществ или тел. В электротехнике эти понятия играют не менее важную роль чем в других разделах физики, ведь от них зависит насколько быстро установка выполнит свою работу и какую нагрузку понесут линии электропередач. Исходя из этих сведений подбираются трансформаторы для подстанций, генераторы для электростанций и сечение проводников передающих линий. В этой статье мы расскажем, как найти мощность электрического прибора или установки, зная силу тока, напряжение и сопротивление.

Определение

Мощность – это скалярная величина. В общем случае она равна отношению выполненной работы ко времени:

P=dA/dt

Простыми словами эта величина определяет, как быстро выполняется работа. Она может обозначаться не только буквой P, но и W или N, измеряется в Ваттах или киловаттах, что сокращенно пишется как Вт и кВт соответственно.

Электрическая мощность равна произведению тока на напряжение или:

P=UI

Как это связано с работой? U – это отношение работы по переносу единичного заряда, а I определяет, какой заряд прошёл через провод за единицу времени. В результате преобразований и получилась такая формула, с помощью которой можно найти мощность, зная силу тока и напряжение.

Формулы для расчётов цепи постоянного тока

Проще всего посчитать мощность для цепи постоянного тока. Если есть сила тока и напряжение, тогда нужно просто по формуле, приведенной выше, выполнить расчет:

P=UI

Но не всегда есть возможность найти мощность по току и напряжению. Если вам они не известны – вы можете определить P, зная сопротивление и напряжение:

P=U2/R

Также можно выполнить расчет, зная ток и сопротивление:

P=I2*R

Последними двумя формулами удобен расчёт мощности участка цепи, если вы знаете R элемента I или U, которое на нём падает.

Для переменного тока

Однако для электрической цепи переменного тока нужно учитывать полную, активную и реактивную, а также коэффициент мощности (соsФ). Подробнее все эти понятия мы рассматривали в этой статье: https://samelectrik.ru/chto-takoe-aktivnaya-reaktivnaya-i-polnaya-moshhnost.html.

Отметим лишь, что чтобы найти полную мощность в однофазной сети по току и напряжению нужно их перемножить:

S=UI

Результат получится в вольт-амперах, чтобы определить активную мощность (ватты), нужно S умножить на коэффициент cosФ. Его можно найти в технической документации на устройство.

P=UIcosФ

Для определения реактивной мощности (вольт-амперы реактивные) вместо cosФ используют sinФ.

Q=UIsinФ

Или выразить из этого выражения:

И отсюда вычислить искомую величину.

Найти мощность в трёхфазной сети также несложно, для определения S (полной) воспользуйтесь формулой расчета по току и фазному напряжению:

S=3UфIф

А зная Uлинейное:

S=1,73*UлIл

1,73 или корень из 3 – эта величина используется для расчётов трёхфазных цепей.

Тогда по аналогии чтобы найти P активную:

P=3UфIф*cosФ=1,73*UлIл*cosФ

Определить реактивную мощность можно:

Q=3UфIф*sinФ=1,73*UлIл*sinФ

На этом теоретические сведения заканчиваются и мы перейдём к практике.

Пример расчёта полной мощности для электродвигателя

Мощность у электродвигателей бывает полезная или механическая на валу и электрическая. Они отличаются на величину коэффициента полезного действия (КПД), эта информация обычно указана на шильдике электродвигателя.

Отсюда берём данные для расчета подключения в треугольник на Uлинейное 380 Вольт:

  1. Pна валу=160 кВт = 160000 Вт
  2. n=0,94
  3. cosФ=0,9
  4. U=380

Тогда найти активную электрическую мощность можно по формуле:

P=Pна валу/n=160000/0,94=170213 Вт

Теперь можно найти S:

S=P/cosφ=170213/0,9=189126 Вт

Именно её нужно найти и учитывать, подбирая кабель или трансформатор для электродвигателя. На этом расчёты окончены.

Расчет для параллельного и последовательного подключения

При расчете схемы электронного устройства часто нужно найти мощность, которая выделяется на отдельном элементе. Тогда нужно определить, какое напряжение падает на нём, если речь идёт о последовательном подключении, или какая сила тока протекает при параллельном включении, рассмотрим конкретные случаи.

Здесь Iобщий равен:

I=U/(R1+R2)=12/(10+10)=12/20=0,6

Общая мощность:

P=UI=12*0,6=7,2 Ватт

На каждом резисторе R1 и R2, так как их сопротивление одинаково, напряжение падает по:

U=IR=0,6*10=6 Вольт

И выделяется по:

Pна резисторе=UI=6*0,6=3,6 Ватта

Тогда при параллельном подключении в такой схеме:

Сначала ищем I в каждой ветви:

I1=U/R1=12/1=12 Ампер

I2=U/R2=12/2=6 Ампер

И выделяется на каждом по:

PR1=12*6=72 Ватта

PR2=12*12=144 Ватта

Выделяется всего:

P=UI=12*(6+12)=216 Ватт

Или через общее сопротивление, тогда:

Rобщее=(R1*R2)/( R1+R2)=(1*2)/(1+2)=2/3=0,66 Ом

I=12/0,66=18 Ампер

P=12*18=216 Ватт

Все расчёты совпали, значит найденные значения верны.

Заключение

Как вы могли убедиться найти мощность цепи или её участка совсем несложно, неважно речь идёт о постоянке или переменке. Важнее правильно определить общее сопротивление, ток и напряжение. Кстати этих знаний уже достаточно для правильного определения параметров схемы и подбора элементов – на сколько ватт подбирать резисторы, сечения кабелей и трансформаторов. Также будьте внимательны при расчёте S полной при вычислении подкоренного выражения. Стоит добавить лишь то, что при оплате счетов за коммунальные услуги мы оплачиваем за киловатт-часы или кВт/ч, они равняются количеству мощности, потребленной за промежуток времени. Например, если вы подключили 2 киловаттный обогреватель на пол часа, то счётчик намотает 1 кВт/ч, а за час – 2 кВт/ч и так далее по аналогии.

Напоследок рекомендуем просмотреть полезное видео по теме статьи:

Также читают:

Как рассчитать мощность электрического тока: формула для расчета по току и напряжению

Пожаловалась бабушка соседка снизу: подарили мне дети моющий пылесос. Он прекрасно работает, но откуда-то идет запах гари.

Пошел смотреть. Проводка у нас старая: лапша из алюминия 2,5 квадрата. А пылесос потребляет 2,5 kW. Прикинул, как работает формула расчета мощности по току и напряжению для этого случая.

Разделил 2500 ватт на 220 вольт. Получил чуть больше 11 ампер. Наши провода держат нагрузку 22 А. Имеем практически двойной резерв по току. Другие потребители при уборке отключены.

Стали проверять и нюхать: запах около квартирного щитка. Открыл, осмотрел: шина сборки ноля в саже, на одной перемычке горелая изоляция. Винт крепления ослаблен. Вот и причина начала возгорания. Исправил.

На этом примере я показываю, что всегда надо оценивать мощность потребления электроприборов и возможности проводки с защитными устройствами. Об этом рассказываю ниже.

Что такое мощность в электричестве: просто о сложном

Вспомнилась былина об Илье Муромце, когда он приложил всю свою мощь к соловью разбойнику. У бедолаги сразу посыпались искры из глаз, как пламя с верхней картинки на проводке с неправильным монтажом.

Простыми словами: мощность в электричестве — это силовая характеристика энергии, которой оценивают, как способности генераторных установок ее вырабатывать, так возможности потребителей и транспортных магистралей.

Все эти участки должны быть точно смонтированы и налажены для обеспечения безопасной работы. Как только в любом месте возникает неисправность, так сразу развивается авария во всей схеме.

Если говорить о домашнем электрическом оборудовании, то приходится постоянно соблюдать баланс между:

  1. включенными в сеть приборами;
  2. конструкцией проводов и кабелей;
  3. настройкой защитных устройств.

Только комплексное решение этих трех вопросов может обеспечить безопасность проводки и жильцов.

Как рассчитать электрическую мощность в быту

Формулы расчета мощности в электричестве позволяют выполнить качественную оценку безопасности каждого из перечисленных выше пунктов.

Пользоваться ими не сложно. Я уже приводил в предыдущих статьях шпаргалку электрика, где они помещены в наглядной форме для цепей постоянного тока.

Они полностью справедливы для активной составляющей мощности переменного тока, совершающей полезную работу. Кстати, кроме нее есть еще и бесполезная — реактивная, связанная с потерями энергии. Ее описанию посвящен второй раздел.

Такие вычисления удобно делать с помощью онлайн калькулятора. Он избавляет от рутинных математических вычислений и арифметических ошибок.

При любом из способов для расчета активной мощности требуется знать две из трех электрических величин:

  1. силу тока I;
  2. приложенное напряжение U;
  3. сопротивление участка цепи R.

Как измерить электрическую мощность дома

Существует еще одна возможность оценки активной мощности: ее измерение в действующей схеме специальными приборами: ваттметрами.

Точные замеры может обеспечить промышленный лабораторный ваттметер. Он изготавливается как прибор, работающий на аналоговых сигналах,так и с помощью цифровых технологий.

В бытовой проводке точные вычисления не нужны. Для нее выпускаются различные виды более простых ваттметров.

Популярностью пользуются приборы, которые можно вставить в розетку и подключить к ним шнур питания от потребителя, включить их в работу и сразу снять показания на дисплее в ваттах.

Их так и называют: ваттметр розетка. Они измеряют чисто активную мощность переменного тока.

Такие приборы избавляют электрика от выполнения сложных операций под напряжением, когда требуется замерять:

  • действующее напряжение;
  • силу тока;
  • угол сдвига фаз между векторами тока и напряжения.

Потом все данные дополнительно требуется вводить в формулу расчета мощности по току и напряжению, делать по ней вычисления.

Этот метод можно упростить, если внимательно наблюдать за показаниями электрического счетчика индукционной системы с вращающимся диском. Он считает совершенную работу: потребленную мощность за определенную время.

Однако скорость вращения диска как раз и характеризует величину потребления. Надо просто посчитать сколько раз он обернется за минуту и перевести в ватты по табличке, расположенной на корпусе.

Почему реактивное сопротивление схемы влияет на мощность переменного тока

Синусоидальная гармоника напряжения, поступая на резистивное сопротивление, изменяет величину тока без его отклонения на комплексной плоскости.

Такой ток совершает полезную работу с минимальными потерями энергии, вырабатывая активную мощность. Частота колебания сигнала не оказывает на нее никакого влияния.

Сопротивление конденсатора и индуктивности зависит от частоты гармоники. Его противодействие отклоняет направление тока на каждом из этих элементов в разные стороны.

Такие процессы связаны с потерей части энергии на бесполезные преобразования. На них расходуется мощность Q, которую называют реактивной.Ее влияние на полную мощность S и связь с активной P удобно представлять графически прямоугольным треугольником.

Захотелось его нарисовать на фоне оборудования из нагромождений фарфора и металла, где пришлось поработать довольно долго.Отвлекся. Не судите за это строго.

Сравните его с опубликованным мною ранее треугольником сопротивлений. Находите общие черты?

Ими являются геометрические пропорции фигуры, описывающие их формулы и угол φ, определяющий потери полной мощности. Перехожу к их более подробному рассмотрению.

Формулы расчета мощности для однофазной и трехфазной схемы питания

В идеальном теоретическом случае трехфазная схема состоит из трех одинаковых однофазных цепей. На практике всегда есть какие-то отклонения. Но, в большинстве случаев при анализах ими пренебрегают.

Поэтому рассматриваем вначале наиболее простой вопрос.

Графики и формулы под однофазное напряжение

Как работает резистор

На чисто резистивном сопротивлении синусоиды тока и напряжения совпадают по углу, направлены на каждом полупериоде одинаково.Поэтому их произведение, выражающее мощность, всегда положительно.

Его значение в произвольный момент времени t называют мгновенным, обозначая строчной буквой p.

Среднее значение мощности в течение одного периода называют активной составляющей. Ее график для переменного тока имеет фигуру симметричного всплеска с максимальным значением Pm в середине каждого полупериода Т/2.

Если взять половину его величины Pm/2 и провести прямую линию в течении одного периода Т, то получим прямоугольник с ординатой P.

Его площадь равна двум площадям графиков активной составляющих одного любого полупериода. Если посмотреть на картинку внимательнее, то можно представить, что верхняя часть всплеска отрезана,перевернута и заполнила свободное пространство внизу.

Представление этого графика помогает запомнить, что на активном сопротивлении мощность постоянного и переменного тока вычисляется по одной формуле, не меняет своего знака.

График мгновенных значений активной мощности переменного тока на резистивном сопротивлении имеет вид повторяющихся положительных волн. Но за один период им совершается такая же работа, как и в цепях постоянного тока и напряжения.

На резисторе не создается реактивных потерь.

Как работает индуктивность

Катушка с обмоткой своими витками запасает энергию магнитного поля. Благодаря процессу ее накопления индуктивное сопротивление отодвигает вперед на 90 градусов вектор тока относительно приложенного напряжения на комплексной плоскости.

Перемножая их мгновенные величины получаем значения мощности, которое за один период меняет знаки (направление) в каждом полупериоде.

Частота изменения мощности на индуктивности в два раза выше,чем у ее составляющих: синусоид тока и напряжения. Она состоит из двух частей:

  1. активной, обозначаемой индексом PL;
  2. реактивной QL.

Реактивная часть на индуктивности создается за счет постоянного обмена энергией между катушкой и приложенным источником. На ее величину влияет значение индуктивного сопротивления XL.

Как работает конденсатор

Емкость конденсатора постоянно накапливает заряд между своими обкладками. За счет этого происходит сдвиг вектора тока вперед на 90 градусов относительно приложенного напряжения.

График мгновенной мощности напоминает вид предыдущего, но начинается с отрицательной полуволны.

Реактивная составляющая, выделяемая на конденсаторе, зависит от величины емкостного сопротивления XC.

Как работает реальная схема со всеми видами сопротивлений

В чистом виде приведенные выше графики и выражения встречаются не так часто. На самом деле передача электроэнергии и ее работа на переменном токе связаны с комплексным преодолением сил электрического сопротивления резисторов, конденсаторов и индуктивностей.

Причем, какая-то из этих составляющих будет преобладать. Для таких случаев преобразования электрической энергии в мгновенную мощность могут иметь один из следующих видов.

На верхней картинке показан случай, когда вектор тока отстает от приложенного напряжения, а на нижней — опережает.

В обоих случаях величина активной составляющей уменьшается от значения полной на значение, выражаемое как cosφ. Поэтому его принято называть коэффициентом мощности.

Косинус фи (cosφ) используется при анализе треугольника мощностей и сопротивлений, характеризует потери энергии.

Как работает схема трехфазного электроснабжения

На ввод распределительного щита многоэтажного здания поступает трехфазное напряжение от электроснабжающей организации, вырабатываемое промышленными генераторами.

Его же, за отдельную плату, при желании может подключить владелец частного дома, что многие и делают. При этом рабочая схема и диаграмма напряжений выглядит следующим образом.

В старой системе заземления TN-C она выполняется четырехпроводным подключением, а у новой TN-S — пятипроводным с добавлением защитного РЕ проводника. Его на этой схеме я не показываю для упрощения.

Каждую из фаз при работе необходимо стараться нагружать одинаково равными по величине токами. Тогда в домашней проводке будет создаваться наиболее благоприятный оптимальный режим без опасных перекосов энергии.

  • В этом случае формула расчета мощности по току и напряжению для трехфазной схемы может быть представлена простой суммой аналогичных формул для составляющих однофазных цепей.
  • А поскольку они все идентичные, то их просто утраивают.
  • Например, когда активная мощность фазы В имеет выражением Рв=Uв×Iв×cosφ, то для всей трехфазной схемы она будет выражена следующей формулой:

Р = Рa+Рв+Рc

Если пометить фазное выражение буквой ф. например Pф, томожно записать:

P = 3Pф = 3Uф×Iф×cosφ

Аналогично будет вычисляться реактивная составляющая

Q = Qa+Qв+Qc

Или

Q = 3Qф = 3Uф×Iф×sinφ

Поскольку P и Q представляют величины катетов прямоугольного треугольника, то гипотенузу или полную составляющую можно вычислить как квадратный корень из суммы их квадратов.

S = √(P2+Q2)

Как учитывается трехфазная полная мощность

В энергосистеме, да и в частном доме, требуется анализировать подключенные нагрузки, равномерно распределять их по источникам напряжений.

С этой целью работают многочисленные конструкции измерительных приборов. На щитах управления подстанций расположены щитовые ваттметры и варметры, предназначенные для работы в разных долях кратности.

Старые аналоговые приборы показаны на этой картинке.

Для того, чтобы не путаться в записях вычислений введены разные наименования единиц. Они обозначаются:

  • ВА — (русское), VA (международное) вольтампер для полной величины мощности;
  • Вт —(русское), var (международное) ватт —активной;
  • вар (русское), var (международное) — реактивной.

Аналоговые приборы измеряют только активную или реактивную составляющую, а полную величину необходимо вычислять по формулам.

Многие современные цифровые приборы способны осуществлять эту функцию автоматически.

Видеоурок Павла Виктор дополняет мой материал. Рекомендую посмотреть.

Калькулятор мощности для своих

Здесь вы можете выполнить вычисления онлайн без использования формул и арифметических действий. Просто введите ваши исходные данные в таблицу и жмите кнопку “Рассчитать ток”.

А в заключение напоминаю, что для ваших вопросов создан раздел комментариев. Задавайте их, я отвечу.

Источник: https://ElectrikBlog.ru/formula_rascheta_moshchnosti_po_toku_i_napryazheniyu_elektroskhemy/

Как рассчитать мощность по току и напряжению?

Любой из элементов электрической сети является материальным объектом определенной конструкции. Но его особенность состоит в двойственном состоянии. Он может быть как под электрической нагрузкой, так и обесточен.

Если электрического подключения нет, целостности объекта ничто не угрожает.

Но при присоединении к источнику электропитания, то есть при появлении напряжения (U) и электротока, неправильная конструкция элемента электросети может стать для него фатальной, если напряжение и электроток приведут к выделению тепла.

Далее из статьи наши читатели получат информацию о том, как правильно сделать расчет мощности по току и напряжению, чтобы электрические цепи работали исправно и продолжительно.

Отличия мощности при постоянном и переменном напряжении

Наиболее простым получается расчет мощности электрических цепей на постоянном электротоке. Для их участков справедлив закон Ома, в котором задействовано только приложенное U, и сопротивление. Чтобы рассчитать силу тока I, U делится на сопротивление R:

  • I=U/R ,
  • причем искомая сила тока именуется амперами.
  • А поскольку электрическая мощность Р для такого случая — это произведение U и силы электротока, она так же легко, как и электроток, вычисляется по формуле:
  • P=U*I ,
  • причем искомая мощность нагрузки именуется ваттами.

Все компоненты этих двух формул характерны для постоянного электротока и называются активными. Напоминаем нашим читателям, что закон Ома, позволяющий выполнить расчет силы тока, весьма многообразен по своему отображению.

Его формулы учитывают особенности физических процессов, соответствующих природе электричества. А при постоянном и переменном U они протекают существенно отличаясь. Трансформатор на постоянном U — это абсолютно бесполезное устройство.

Также как синхронные и асинхронные движки.

Принцип их функционирования заключен в изменяющемся магнитном поле, создаваемом элементами электрических цепей, обладающими индуктивностью. А такое поле появляется только как следствие переменного U и соответствующего ему переменного тока.

Но электричеству свойственно также и накопление зарядов в элементах электрических цепей. Это явление называется электрической емкостью и лежит в основе конструкции конденсаторов.

Параметры, связанные с индуктивностью и емкостью, называют реактивными.

Расчет мощности в цепях переменного электротока

Поэтому, чтобы определить ток по мощности и напряжению как в обычной электросети 220 В, так и в любой другой, где используется переменное U, потребуется учесть несколько активных и реактивных параметров.

Для этого применяется векторное исчисление. В результате отображение рассчитываемой мощности и U имеет вид треугольника. Две стороны его — это активная и реактивная составляющие, а третья — их сумма.

Например, полная мощность нагрузки S, именуемая вольт-амперами.

Реактивная составляющая называется варами. Зная величины сторон для треугольников мощности и U, можно выполнить расчет тока по мощности и напряжению. Как это сделать, поясняет изображение двух треугольников, показанное далее.


Треугольники мощности и напряжения

Для измерения мощности применяются специальные приборы. Причем их многофункциональных моделей совсем мало.

Это связано с тем, что для постоянного электротока, а также в зависимости от частоты используется соответствующий конструктивный принцип измерителя мощности.

По этой причине прибор, предназначенный для измерения мощности в цепях переменного электротока промышленной частоты, на постоянном электротоке или на повышенной частоте будет показывать результат с неприемлемой погрешностью.


Лабораторный ваттметр


Щитовой ваттметр

У большинства наших читателей выполнение того или иного вычисления с использованием величины мощности скорее всего происходит не с измеренным значением, а по паспортным данным соответствующего электроприбора.

При этом можно легко рассчитать ток для определения, например, параметров электропроводки или соединительного шнура. Если U известно, а оно в основном соответствует параметрам электросети, расчет тока по мощности сводится к получению частного от деления мощности и U.

Полученный таким способом расчетный ток определит сечение проводов и тепловые процессы в электрической цепи с электроприбором.  

Но вполне закономерен вопрос, как рассчитать ток нагрузки при отсутствии каких-либо сведений о ней? Ответ следующий. Правильный и полный расчет тока нагрузки, запитанной переменным U, возможен на основании измеренных данных.

Они должны быть получены с применением прибора, который замеряет фазовый сдвиг между U и электротоком в цепи. Это фазометр. Полный расчет мощности тока даст активную и реактивную составляющие.

Они обусловлены углом φ, который показан выше на изображениях треугольников.


Лабораторный фазометр


Щитовой фазометр

Используем формулы

Этот угол и характеризует фазовый сдвиг в цепях переменного U, содержащих индуктивные и емкостные элементы. Чтобы рассчитывать активные и реактивные составляющие, используются тригонометрические функции, применяющиеся в формулах. Перед тем как посчитать результат по этим формулам, надо, используя калькуляторы или таблицы Брадиса, определить sin φ и cos φ. После этого по формулам

я вычислю искомый параметр электрической цепи.

Но следует учесть то, что каждый из параметров, рассчитанный по этим формулам, из-за U, постоянно изменяющегося по законам гармонических колебаний, может принимать либо мгновенное, либо среднеквадратичное, либо промежуточное значение.

Три формулы, показанные выше, справедливы при среднеквадратичных значениях силы электротока и U. Каждое из двух остальных значений является результатом расчетной процедуры с использованием другой формулы, учитывающей ход времени t:

Но и это еще не все нюансы. Например, для линий электропередачи применяются формулы, в которых фигурируют волновые процессы. И выглядят они по-другому. Но это уже совсем другая история…  

Источник: https://domelectrik.ru/elektrosnabzhenie/bezopasnost/raschet-moshchnosti

Формула мощности электрического тока, расчет по мощности и напряжению

Для того, чтобы обеспечить безопасность при эксплуатации промышленных и бытовых электрических приборов, необходимо правильно вычислить сечение питающей проводки и кабеля. Ошибочный выбор сечения жил кабеля может привести из-за короткого замыкания к возгоранию проводки и к возникновению пожара в здании.

Что такое мощность (Р) электротока

Электрическая мощность является физической величиной, характеризующей скорость преобразования или передачи электрической энергии. Единицей измерения по Международной системе единиц (СИ) является ватт, в нашей стране обозначается Вт, международное обозначение — W.

Что влияет на мощность тока

На мощность (Р) влияет величина силы тока и величина приложенного напряжения. Расчет параметров электроэнергии выполняется еще на стадии проектирования электрических сетей объекта.

Полученные данные позволяют правильно выбрать питающий кабель, к которому будут подключаться потребители. Для расчетов силы электротока используется значения напряжения сети и полной нагрузки электрических приборов.

В соответствии с величиной силы электротока выбирается сечение жил кабелей и проводов.

Отличия мощности при постоянном и переменном напряжении

Ведем обозначения электрических величин, которые приняты в нашей стране:

  • Р − активная мощность, измеряется в ваттах, обозначается Вт;
  • Q − реактивная мощность, измеряется в вольт амперах реактивных, обозначается ВАр;
  • S − полная мощность, измеряется в вольт амперах, обозначается ВА;
  • U − напряжение, измеряется в вольтах, обозначается ВА;
  • I − ток, измеряется в амперах, обозначается А;
  • R − сопротивление, измеряется в омах, обозначается Ом.

Назовем основные отличия P на постоянном и Q на переменном электротоке. Расчет P на постоянном электротоке получается наиболее простым. Для участков электрической цепи справедлив закон Ома. В этом законе задействованы только величина приложенного U (напряжения) и величина сопротивления R.

Расчет S (полной мощности) на переменном электротоке производится несколько сложнее. Кроме P, имеется Q и вводится понятие коэффициента мощности. Алгебраически складывая активную P и реактивную Q, получают общую S.

По какой формуле вычисляется

Расчет силы тока по мощности и напряжению в сети постоянного тока

Для расчета силы I (тока), надо величину U (напряжения) разделить на величину сопротивления.

  • Расчет силы тока по мощности и напряжению:
  • I = U ÷ R
  • Измеряется в амперах.

Для такого случая электрическую Р (активную мощность) можно посчитать как произведение силы электрического I на величину U.

Формула расчета мощности по току и напряжению:

P = U × I

Все компоненты в этих двух формулах характерны для постоянного электротока и их называют активными.

  1. Исходя из этих двух формул, можно вывести также еще две формулы, по которым можно узнавать P:
  2. P = I2 × R
  3. P = U2 ÷ R

Однофазные нагрузки

В однофазных сетях переменного электротока требуется произвести вычисление отдельно для Р и Q нагрузки, затем надо при помощи векторного исчисления их сложить.

В скалярном виде это будет выглядеть так:

S = √P2 + Q2

В результате расчет P, Q, S имеет вид прямоугольного треугольника. Два катета этого треугольника представляют собой P и Q составляющие, а гипотенуза — их алгебраическую сумму.

S измеряется в вольт-амперах (ВА), Q измеряется в вольт-амперах-реактивных (ВАр), Р измеряется в ваттах (Вт).

Зная величины катетов для треугольников, можно рассчитать коэффициент мощности (cos φ). Как это сделать, показано на изображении треугольника.

Расчет в трехфазной сети

Переменный I (ток) отличается от постоянного по всем параметрам, особенно наличием нескольких фаз. Расчет P в трехфазной нагрузке необходим для правильного определения характеристик подключаемой нагрузки. Трехфазные сети широко применяются в связи с удобством эксплуатации и малыми материальными затратами.

Трехфазные цепи могут соединяться двумя способами – звездой и треугольником. На всех схемах фазы обозначают символами А, В, С. Нейтральный провод обозначают символом N.

При соединении звездой различают два вида U (напряжения) – фазное и линейное. Фазное U определяется как U между фазой и нейтральным проводом. Линейное U определяется как U между двумя фазами.

Эти два U связаны между собой соотношением:

UЛ = UФ × √3

Линейные и фазные электротоки при соединении звездой равны друг другу: IЛ = IФ

Форма расчета S при соединении звездой:

S = SA + SB + SC = 3 × U × I

Активная P:

Р = 3 × Uф × Iф × cosφ

Реактивная Q:

Q = √3 × Uф × Iф × sinφ.

При соединении треугольником фазное и линейное U равны друг другу: UЛ = UФ

Линейный I при соединении треугольником определяется по формуле:

IЛ = IФ × √3

Формулы мощности электрического тока при соединении треугольником:

  • S = 3 × Sф = √3 × Uф × Iф;
  • Р = √3 × Uф × Iф × cosφ;
  • Q = √3 × Uф × Iф × sinφ.

Средняя P в активной нагрузке

В электрических сетях P измеряют при помощи специального прибора – ваттметра. Схемы подключения находятся в зависимости от способа подключения нагрузки.

При симметричной нагрузке P измеряется в одной фазе, а полученный результат умножают на три. В случае несимметричной нагрузки для измерения потребуется три прибора.

Параметры P электросети или установки являются важными данными электрического прибора. Данные по потреблению P активного типа передаются за определенный период времени, то есть передается средняя потребляемая P за расчетный период времени.

Подбор номинала автоматического выключателя

Автоматические выключатели защищают электрические аппараты от токов короткого замыкания и перегрузок.

При аварийном режиме они обесточивают защищаемую цепь при помощи теплового или электромагнитного механизма расцепления.

Тепловой расцепитель состоит из биметаллической пластины с различными коэффициентами теплового расширения. Если номинальный ток превышен, пластина изгибается и приводит в действие механизм расцепления.

У электромагнитного расцепителя имеется соленоид с подвижным сердечником. При превышении заданного I, в катушке увеличивается электромагнитное поле, сердечник втягивается в катушку соленоида, в результате чего срабатывает механизм расцепления.

  • Минимальный I, при котором тепловой расцепитель должен сработать, устанавливается с помощью регулировочного винта.
  • Ток срабатывания у электромагнитного расцепителя при коротком замыкании равен произведению установленного срабатывания на номинальный электроток расцепителя.

Видео о законах электротехники

Из следующего видео можно узнать, что такое электричество, мощность электрического тока. Даны примеры практического применения законов электротехники.

Источник: https://vdome.club/materialy/raschety/formula-moschnosti.html

Мощность ток напряжение. Расчёт нагрузки и выбор питающих кабелей

Электроэнергия давно используется человеком для удовлетворения своих потребностей, но она невидима, не воспринимается органами чувств, потому сложна для понимания. Мощность ток напряжение, все эти характеристики электроэнергии исследованы известными учеными, которые дали им определения и описали математическими методами взаимные связи между ними.

   Мощность ток напряжение сопротивление 

Так же следует помнить, на величину электрического сопротивления влияет несколько факторов:

  • строение вещества, определяющее наличие свободных электронов в проводнике и влияющее на удельное сопротивление
  • площадь поперечного сечения и длина токовода
  • температура

В приведенной таблице показаны общие соотношения для цепей постоянного и переменного тока, которые можно применять для анализа работы схем электроснабжения. 

Расчёт сечения питающего кабеля и проводки

Для обеспечения безопасности при эксплуатации бытовых электроприборов необходимо верно вычислить сечение питающего кабеля и проводки. Поскольку ошибочно выбранное сечение жил кабеля способно привести к перегреву провода, плавление его изоляции и в итоге, возгоранию, из-за короткого замыкания. 

   Мощность ток напряжение, удобная шпаргалка

Основным параметром, по которому производят расчет сечения провода, является его продолжительная допустимая токовая нагрузка. Т.

е, это такая номинальная величина тока, которую проводник способен через себя пропускать на протяжении длительного времени.

Для определения величины номинального тока, необходимо знать приблизительную мощность всех подключаемых электроприборов и оборудования в квартире.

И так, что мы имеем:

  • От значения величины тока зависит выбор питающего кабеля (провода), по которому могут быть подключены приборы энергопотребления к сети
  • Зная напряжение электрической сети и полную нагрузку электроприборов, можно по формуле вычислить силу тока, который потребуется пропускать по проводнику(проводу, кабелю). По его величине выбирают площадь сечения жил.

Расчет тока, выполняем самостоятельно

Если известны электро-потребители в квартире или доме, необходимо выполнить несложные расчёты, чтобы правильно смонтировать схему электроснабжения.

Аналогичные расчёты выполняются для производственных целей: определения необходимой площади сечения жил кабеля при осуществлении подключения промышленного оборудования (различных промышленных электрических двигателей и механизмов).

Мощность ток напряжение, расчёты для однофазной сети 220 В

Сила тока I (в амперах, А) подсчитывается по формуле:

  • I = P / U,
  • где
  • P – электрическая полная нагрузка (обязательно указывается в техническом паспорте устройства), Вт (ватт)
  • U – напряжение электрической сети, В (вольт)

Ниже в таблице представлены величины нагрузки типичных бытовых электроприборов и потребляемый ими ток (для напряжения 220 В).

ЭлектроприборПотребляемая мощность, ВтСила тока, А
Стиральная машина2000 – 25009,0 – 11,4
Джакузи2000 – 25009,0 – 11,4
Электроподогрев пола800 – 14003,6 – 6,4
Стационарная электрическая плита4500 – 850020,5 – 38,6
СВЧ печь900 – 13004,1 – 5,9
Посудомоечная машина2000 — 25009,0 – 11,4
Морозильники, холодильники140 — 3000,6 – 1,4
Мясорубка с электроприводом1100 — 12005,0 — 5,5
Электрочайник1850 – 20008,4 – 9,0
Электрическая кофеварка6з0 — 12003,0 – 5,5
Соковыжималка240 — 3601,1 – 1,6
Тостер640 — 11002,9 — 5,0
Миксер250 — 4001,1 – 1,8
Фен400 — 16001,8 – 7,3
Утюг900 — 17004,1 – 7,7
Пылесос680 — 14003,1 – 6,4
Вентилятор250 — 4001,0 – 1,8
Телевизор125 — 1800,6 – 0,8
Радиоаппаратура70 — 1000,3 – 0,5
Приборы освещения20 — 1000,1 – 0,4

Различные потребители электроэнергии подключаются через соответствующие автоматы к электросчётчику и далее общему автомату, который должен быть рассчитан на нагрузку приборов, которыми будет оборудована квартира. Провод, который подводит питание также должен удовлетворять нагрузке энергопотребителей.

Как рассчитать ток защитного автомата

Для группы розеток, предназначенных для питания бытовых электроприборов на кухне, необходимо подобрать защитный автоматический выключатель. Мощности приборов по паспортным данным составляют 2,0, 1,5 и 0,6 кВт.

Решение. В квартире используется однофазная переменная сеть 220 вольт. Общая мощность всех приборов, подключенных в работу одновременно, составит 2,0+1,5+0,6=4,1 кВт=4100 Вт.

По формуле I = P / U определим общий ток группы потребителей: 4100/220=18,64 А.

Ближайший по номиналу автоматический выключатель имеет величину срабатывания 20 ампер. Его и выбираем. Автомат меньшего значения на 16 А будет постоянно отключаться от перегрузки.

Ниже приводится таблица для скрытой проводки при однофазной схеме подключения квартиры для подбора провода при напряжении 220 В

Сечение жилы провода, мм2Диаметр жилы проводника, ммМедные жилыАлюминиевые жилы
Ток, АМощность, ВтТок, АМощность, кВт
0,500,8061300
0,750,98102200
1,001,13143100
1,501,38153300102200
2,001,60194200143100
2,501,78214600163500
4,002,26275900214600
6,002,76347500265700
10,003,575011000388400
16,004,5180176005512100
25,005,64100220006514300

Как видно из таблицы сечение жил зависит кроме нагрузки и от материала, из которого изготовлен провод.

Мощность ток напряжение, расчёты для трёхфазной сети 380 В

При трёхфазном электроснабжении сила тока I (в амперах, А) вычисляется по формуле:

  1. I = P /1,73 U,
  2. где P -потребляемая мощность, Вт;
  3. U — напряжение в сети, В,
  4. так как напряжение при трёхфазной схеме электроснабжения 380 В, формула примет вид:
  5. I = P /657, 4.

Сечение жил в питающем кабеле при различной нагрузке при трёхфазной схеме напряжением 380 В для скрытой проводки представлена в таблице.

Сечение жилы провода, мм2Диаметр жилы проводника, ммМедные жилыАлюминиевые жилы
Ток, АМощность, ВтТок, АМощность, кВт
0,500,8062250
0,750,98103800
1,001,13145300
1,501,38155700103800
2,001,60197200145300
2,501,78217900166000
4,002,262710000217900
6,002,763412000269800
10,003,5750190003814000
16,004,5180300005520000
25,005,64100380006524000

Для расчёта тока в цепях питания нагрузки, характеризующейся большой реактивной полной мощностью, что характерно применению электроснабжения в промышленности:

  • электрические двигатели
  • дроссели приборов освещения
  • сварочные трансформаторы
  • индукционные печи

В мощных приборах и оборудовании, доля реактивной нагрузки выше и поэтому для таких приборов в расчетах коэффициент мощности принимают равным 0,8.

На практике принято считать, что при подсчёте электрических нагрузок для бытовых целей запас мощности принимают 5%. В случае расчёта электрических сетей для промышленного производства запас мощности принимают 20%.

Будем рады, если подпишетесь на наш Блог!

[wysija_form id=»1″]
  • ТЕГИ
  • Монтаж кабеля
  • Предварительный монтаж
  • Схемы

Источник: https://powercoup.by/stati-po-elektromontazhu/moshhnost-tok-napryazhenie

Расчет мощности по току и напряжению, схема и таблицы

Главная страница » Ремонт квартиры » Электрика » Расчет мощности по току и напряжению, схема и таблицы.

Расскажите друзьям и коллегам в социальных сетях!

Чтобы обезопасить себя при работе с бытовыми электроприборами, необходимо в первую очередь правильно вычислить сечение кабеля и проводки. Потому-что если будет неправильно выбран кабель, это может привести к короткому замыканию, из за чего может произойти возгорание в здание, последствия могут быть катастрофическими.

Это правило относиться и к выбору кабеля для электродвигателей.

Расчёт мощности по току и напряжению

Данный расчет происходит по факту мощности, проделывать его необходимо еще до начала проектирование своего жилища (дома, квартиры).

  • Из этого значение  зависят кабеля питающие приборы которые подключены к электросети.
  • По формуле можно вычислить силу тока, для этого понадобиться взять точное напряжение сети и нагрузку питающихся приборов. Ее величина дает нам понять площадь сечение жил.

Если вам известны все электроприборы, которые в будущем должны питаться от сети, тогда можно легко сделать расчеты для схемы электроснабжение. Эти же расчеты можно выполнять и для производственных целей.

Однофазная сеть напряжением 220 вольт
  • Формула силы тока I (A — амперы):
  • I=P/U
  • Где P — это электрическая полная нагрузка (ее обозначение обязательно указывается в техническом паспорте данного устройства), Вт — ватт;
  • U — напряжение электросети, В (вольт).
  • В таблице представлены стандартные нагрузки электроприборов и потребляемый ими ток (220 В).
  • На рисунке вы можете видет схему устройства электроснабжение дома при однофазном подключении к сети 220 вольт.

Схема приборов при однофазном напряжении

Как и показано на рисунке, все потребители должны быть подключены к соответствующим автоматам и счетчику, далее к общему автомату который будет выдерживать общею нагрузку дома. Кабель который будет доводит ток, должен выдерживать нагрузку всех подключенных бытовых приборов.

В таблице ниже показана скрытая проводка при однофазной схеме подключение жилища для подбора кабеля при напряжении 220 вольт.

Как и показано в таблице, сечение жил зависит и от материала из которого изготовлен.

Трёхфазная сеть напряжением 380 В
  1. В трехфазном электроснабжении сила тока рассчитывается по следующей формуле:
  2. I = P /1,73 U
  3. P — потребляемая мощность в ватах;
  4. U — напряжение сети в вольтах.
  5. В техфазной схеме элетропитания 380 В, формула имеет следующий вид:
  6. I = P /657, 4
  7. Если к дому будет проводиться трехфазная сеть 380 В, то схема подключения будет иметь следующий вид.
  8. В таблице ниже представлена схема сечения жил в питающем кабеле при различной нагрузке при трехфазном напряжении 380 В для скрытой проводки.
  9. Для дальнейшего расчета питания в цепях нагрузки, характеризующейся большой реактивной полной мощностью, что характерно применению электроснабжения в промышленности:
  • электродвигатели;
  • индукционные печи;
  • дроссели приборов освещения;
  • сварочные трансформаторы.

Это явление в обязательном порядке необходимо учитывать при дальнейших расчетах. В более мощных электроприборах нагрузка идет гораздо больше, поэтому в расчетах коэффициент мощности принимают 0,8.

При подсчете нагрузки на бытовые приборы запас мощности нужно брать 5%. Для электросети этот процент становит 20%.

Оцените качество подачи материала: (3

Источник: https://DomStrouSam.ru/raschet-moshhnosti-po-toku-i-napryazheniyu-shema-i-tablitsyi/

Расчет мощности по току и напряжению: 3 способа | Мое мнение: ремонт

Грамотный электрик никогда не станет делать монтаж без расчета оптимальных условий работы оборудования, проводки и анализа возможностей ликвидации случайно возникающих аварий.

Расчет мощности по току и напряжению обеспечивает безопасность оборудования и защиту здоровья жильцов всей схемы питания, начиная от ввода в квартиру или частный дом и заканчивая конечным потребителем.

Все эти места необходимо грамотно просчитать. Выполнить это можно одним из трех нижеприведенных способов. Но лучше — комбинацией двух из них, которая исключит случайную ошибку.

Краткие сведения: что такое электрическая мощность

В электротехнике термин мощности используется для определения силовых, энергетических характеристик источников напряжения, нагрузок и потребителей.

В цепях постоянного тока ее рассчитывают элементарным перемножением силы тока на приложенное напряжение. Для бытовой проводки действуют эти же простые закономерности. Однако промышленная частота 50 герц накладывает свои особенности на вычисления.

Здесь действуют свои законы и термины: активная и реактивная составляющие полной мощности.

Нагрузки типа ламп накаливания и ТЭН обладают только резистивным сопротивлением и не отклоняют вектор тока от напряжения, их называют активными.

График активной мощности P по времени постоянен и не изменяется. Реактивная составляющая отсутствует.

Потребление активной мощности бытовыми приборами удобно замерять доступными по цене ваттметр-розетками.

Реактивную составляющую мощности формируют нагрузки емкостного и индуктивного характеров. На индуктивностях, а это обмотки электродвигателей, трансформаторов, дросселей ток начинает отставать от вектора напряжения.

Реактивная составляющая индуктивной мощности обозначается индексом QL.

На емкости, а это конденсаторная нагрузка, ток начинает опережать напряжение, забегает вперед на 90 угловых градусов.

Однако в чистом виде в нашей проводке эти нагрузки не проявляются, а работают в комплексе. Примерно так.

Причем для их расчета удобно использовать треугольник мощностей. Им пользуются во всех системах напряжения: от высоковольтных до радиолюбительских.

В быту мы платим деньги за потребление только активной мощности по счетчику.

Реактивные составляющие могут считать только специально предназначенные для этого приборы.

Однако переходим к рассмотрению трех способов того, как можно просто выполнить расчет мощности по току и напряжению своими руками.

Способ №1: расчет по формулам

Здесь все просто и понятно. Особо расписывать нечего. Давно уже действует шпаргалка электрика, которая вобрала в себя все основные формулы соотношений между током, напряжением, мощностью и сопротивлением.

Она справедлива для всех случаев жизни, отлично работает в цепях постоянного тока. Для переменки тоже действует, но формулы следует применять не чисто в арифметическом виде, а используя геометрические выражения, комплексные числа или другие методы высшей математики.

Способ №2: онлайн-калькулятор расчета мощности

Здесь приводить его сложно. Поэтому переходите по ссылке на мой блог электрика.

Онлайн калькулятор расчета мощности расположен по этой ссылке. Расписывать не буду. Просто вводите исходные данные и получаете сразу готовый результат.

Программа Электрик 7.8: полезный ресурс для всех начинающих мастеров

Если первые два способа по каким-то причинам вас не устроили, то рекомендую воспользоваться полезной компьютерной программой.

Она создана специально для начинающих мастеров и позволяет выполнять расчеты даже средней сложности в домашней проводке. Называется она Программа Электрик 7.8.

Основной недостаток ее состоит в том, что она размещена на бесплатном хостинге, что из-за обилия рекламы затрудняет немного ее скачивание. Но все это преодолимо.

Вам надо в поисковую строку любого браузера ввести адрес, выделенный на картинке. После этого будет дана возможность скачать и установить программу на свой компьютер.

Весь этот процесс я здесь описывать не стану. Он подробно со скриншотами представлен на моем сайте. Заходите и смотрите, кому надо.

После этого вы сможете не только делать расчет мощности, но легко получать такие вот схемы с подробными рекомендациями, как у меня после ввода исходных данных.

Для любителей работать по видеороликам рекомендую к просмотру видео на канале Ютуб владельца Заметки электрика.

Напоминаю, что вам сейчас по горячим следам удобно задать вопрос, оценить статью и поделиться ею с друзьями в соц сетях.

Калькулятор мощности переменного тока • Электротехнические и радиотехнические калькуляторы • Онлайн-конвертеры единиц измерения

По этим трехфазным высоковольтным линиям электропередачи передается электроэнергия, выработанная на АЭС Пикеринг, расположенной на оз. Онтарио в 13 км от Торонто. Высокое напряжение используется для повышения эффективности передачи электроэнергии в результате уменьшения тепловых потерь в проводах.

Определения и формулы

Этот калькулятор используется для расчета мощности переменного тока и все, о чем говорится ниже, относится к переменному току. Если вы хотите рассчитать мощность по постоянному току, воспользуйтесь нашим Калькулятором мощности постоянного тока. В описании этого калькулятора вы найдете информацию о фундаментальных понятиях электротехники: заряде, силе тока, напряжении и мощности, а также о единицах их измерения. Здесь мы рассмотрим расчет электрической мощности в однофазной сети переменного тока.

В отличие от постоянного тока, который течет только в одном направлении, переменный ток периодически изменяет направление и амплитуду с течением времени. Следовательно, этот калькулятор, который считает мощность переменного тока, значительно сложнее калькулятора мощности постоянного тока. Вместо просто мощности постоянного тока в схемах постоянного тока, здесь мы будем говорить сразу о четырех видах мощности: активной мощности, P, реактивной мощности, Q, комплексной мощности, S, and полной мощности, |S|. Похоже, что четыре мощности вместо одной — слишком сложно? Ничего, мы попробуем разобраться.

Переменный ток

Установленный на столбе в жилой зоне в Канаде однофазный распределительный трансформатор, подающий потребителю ток напряжением 120 V.

Переменный ток может быть не только синусоидальной формы, как в сетевых розетках. Он может иметь любую форму, в том числе и не периодическую. Примером такой сложной формы может быть звук гитарной струны, в которой одновременно возникают колебания нескольких собственных частот струны. В результате кажется, что одновременно слышен звук нескольких частот. Однако, в описании этого калькулятора мы будем говорить только о синусоидальных колебаниях.

Для уменьшения тепловых потерь в проводах линий электропередачи, которые переносят энергию от электростанции потребителям, используется высокое напряжение до сотен киловольт. Это высокое напряжение преобразуется в более безопасное напряжение 110 или 220 В. Использовать высокое напряжение без понижения было бы очень неудобно и опасно.

Исторически сложилось так, что частота электросетей в разных странах различная, причем чаще всего встречаются частоты 50 и 60 Гц. В морской, авиационной и космической технике используется частота 400 Гц, так как она позволяет уменьшить вес оборудования, такого как трансформаторы и электродвигатели, а также увеличить скорость работы электродвигателей. Однако такая высокая частота неудобна для передачи на большие расстояния, так как в результате значительно увеличивается импеданс линий электропередачи из-за их индуктивности.

Подробнее об электрическом токе — в нашем Конвертере электрического тока.

Напряжение

Мгновенное напряжение u(t) представляется функций времени:

где Up — пиковое значение напряжения (максимальная амплитуда) в вольтах, ω — угловая частота в радианах в секунду и f — частота в герцах. Для описания напряжения используется также величина размаха напряжения или двойная амплитуда (англ. peak-to-peak amplitude) Upp = 2Up. Здесь мы используем для обозначения напряжения нижний регистр u(t), чтобы показать, что это выражение для изменения мгновенного напряжения в зависимости от времени t.

Величиной размаха напряжения удобно пользоваться, например, при оценке максимального пробивного напряжения изоляции и конденсаторов. В то же время, размахом напряжения пользоваться неудобно, если нужно оценить мощность переменного тока. В этом случае удобно использовать действующее (среднеквадратичное, англ. root mean square, RMS) значение напряжения, так как такое напряжение нагревает чисто резистивную нагрузку точно так же, как это делает постоянный ток с тем же напряжением. Например, если действующее значение напряжения 220 В приложено к идеальному резистору, на нем выделится столько же тепла, сколько выделилось бы если бы к нему было приложено постоянное напряжение 220 В. Новые микропроцессорные мультиметры обычно измеряют действительное среднеквадратичное значение напряжение сигнала любой формы, так как они оцифровывают сигнал, то есть, преобразуют его в набор дискретных выборок, а затем рассчитывают среднеквадратичное значение напряжения.

Соотношение между действующим (RMS) и амплитудным значением (А) для часто используемых периодических функций хорошо известно и получено в результате интегрирования одного периода этих функций по времени:

  • синусоидальные колебания:

  • прямоугольные импульсы (меандр) со скважностью (отношение периода к длительности импульса) 50%:

  • прямоугольные импульсы со скважностью D:

  • треугольные импульсы:

Подробную информацию о напряжении можно найти в нашем Конвертере электрического потенциала и напряжения

Мощность

В типичной цепи переменного тока энергия передается по линии электропередачи от источника, например, электростанции или портативного генератора, к нагрузке, например, к лампе или телевизору. Поскольку соединительные провода имеют небольшое сопротивление, часть энергии расходуется на нагрев этих проводов и затем на нагрев окружающей среды. Бóльшая часть энергии передается в нагрузку. Если нагрузка резистивная, энергия преобразуется в тепловую и нагревает окружающую среду. Если нагрузка резистивно-индуктивная, например, электродвигатель, то электрическая энергия вначале преобразуется в механическую плюс тепловую (двигатель нагревается) и в дальнейшем вся она преобразуется в тепловую и опять же нагревает окружающую среду.

Электрическая мощность P представляет собой скорость передачи энергии в нагрузку или ее преобразования:

Здесь U — напряжение в вольтах, I — ток в амперах. В Европейских странах для обозначения напряжения обычно используют букву U. В Северной Америке для обозначения напряжения обычно используют V, потому что V — сокращение для вольта. Конечно, это неудобно, но все привыкли, так же как к фунтам, футам и дюймам. Сравните: V = 1 V и U = 1 V. Что удобнее?

Из закона Ома мы знаем, что

Поэтому мощность на резистивной нагрузке можно выразить как

где R — сопротивление в омах. В нашем Конвертере единиц мощности, описано, что мощность измеряется в ваттах (Вт). Процесс преобразования электрической энергии в тепловую обычно называется джоулевым нагревом.

Для установившегося синусоидального сигнала мгновенное напряжение u с фазовым углом φu и мгновенный ток i с фазовым углом φi можно выразить в виде

Для удобства мы предположим, что φi = 0, когда ток проходит положительный максимум. Тогда разность фаз между током и напряжением становится равной просто φu. Теперь можно преобразования функции для тока и напряжения к виду

Мгновенная мощность определяется произведением тока и напряжения

Преобразуем эту формулу, используя тригонометрическое тождество для произведения двух косинусов:

Теперь воспользуемся тригонометрическим тождеством для косинуса суммы двух аргументов:

Мгновенное напряжение, ток и мощность чистого синусоидального процесса в индуктивной нагрузке. Ток в индуктивной нагрузке отстает от напряжения (φu = 60°) и, следовательно, в данном случае мы имеем «отстающий» коэффициент мощности или cos φ = 0,5. Отрицательная часть красной синусоиды функции мощности под горизонтальной осью показывает часть мощности, которая возвращается в систему

На рисунке выше показано соотношение между мгновенными значениями напряжения, тока и мощности в индуктивной нагрузке в предположении, что фазовый сдвиг φu = 60°.

Для чисто резистивной нагрузки мощность определяется так:

или

Среднеквадратичное значение называют также эффективным значением синусоидального тока или напряжения.

Активная и реактивная мощность

Мы можем переписать формулу для мгновенной мощности в виде

или

где величина

называется активной, P. Это часть полной мощности, которая преобразуется в нагрузке в тепло и другие виды энергии и измеряется в ваттах (Вт). Величина

называется реактивной мощностью, Q. Это часть полной мощности, которая в течение каждого цикла возвращается к источнику энергии и измеряется в реактивных вольт-амперах (вар). Эту единицу можно использовать с десятичными приставками для образования дольных и кратных единиц, например, мвар, квар, Мвар (мегавар), ТВА (теравар), ГВА (гигавар) и т. д.

Можно преобразовать выражение для активной и реактивной мощности с использованием среднеквадратичных значений напряжения и тока:

Конечно, в реальной жизни все нагрузки не только резистивные, но также емкостные или индуктивные. Даже электронагреватель имеет определенные емкость и индуктивность (спираль — катушка индуктивности, а отдельные витки образуют конденсаторы). Трансформаторы и электродвигатели являются примерами индуктивных нагрузок. Конденсаторы и катушки индуктивности запасают энергию во время протекания в них переменного тока, в результате чего направление передачи энергии в цепи периодически изменяется. В цепи переменного тока с чисто резистивной нагрузкой синусоидальные ток и напряжение изменяют полярность одновременно, поэтому направление передачи энергии не изменяется и передается только активная энергия.

Если нагрузка чисто реактивная (индуктивная или емкостная), то разность фаз между напряжением и током равна 90° (подробнее об этом поведении RLC цепей). В этом случае энергия в нагрузку вообще не передается. В то же время, электроэнергия течет от источника в нагрузку и возвращается назад по линиям электропередачи, которые в результате нагреваются и нагревают окружающую среду. В связи с тем, что реальные нагрузки всегда имеют некоторую индуктивность и емкость, в них всегда имеется активная и реактивная составляющие мощности.

Комплексная и полная мощность

Возможно для того чтобы всё усложнить, а может быть, наоборот, чтобы упростить, инженеры придумали еще два вида мощности: комплексную мощность, S, измеряемую в вольт-амперах (ВА) и полную мощность, |S|, которая является векторной суммой активной и реактивной мощностей и также измеряется в вольт-амперах. Эту единицу можно использовать с десятичными приставками для образования дольных и кратных единиц, например, мВА, кВА, МВА (мегавольт-ампер), ТВА (теравольт-ампер), ГВА (гигавольт-ампер) и т. д.

Комплексная мощность, S — комплексная сумма активной и реактивной мощностей:

Мы увидим, что комплексная мощность объединяет активную и реактивную мощности, а также коэффициент мощности.

Полная мощность, |S| — модуль (абсолютная величина) комплексной мощности:

Треугольник мощностей показывает комплексную мощность, которая является векторной суммой активной P и реактивной Q мощностей; полная мощность |S| является абсолютной величиной (модулем) комплексной мощности.

Из треугольника мощностей имеем:

Используя тригонометрическое тождество, являющееся следствием теоремы Пифагора и приведенные выше формулы для P и Q, можно записать:

То есть, полная мощность |S| является произведением действительных значений напряжения и тока.

Комплексная мощность учитывается при разработке и эксплуатации энергетических систем, потому что линии электропередач, трансформаторы и генераторы должны быть рассчитаны на полную мощность, а не только на мощность, которая выполняет полезную работу. Если реактивной мощности недостаточно, это может привести к понижению напряжения и даже, в свою очередь, к большой аварии в электросистеме (блэкауту), например, такой, как авария в энергосистеме США и Канады в 2003 году, в результате которой 55 миллионов человек на северо-западе США и в канадской провинции Онтарио остались без электроэнергии.

Электродвигателя являются примерами индуктивных промышленных нагрузок

Коэффициент мощности

Коэффициент мощности определяется как отношения реальной (активной) мощности, поглощенной нагрузкой P к полной мощности |S| в системе. В русскоязычной литературе коэффициент мощности обычно обозначается λ (в процентах) или cos φ, где φ — угол сдвига фаз между током и напряжением. В этой статье, поскольку она является переводом с английского без изменения формул, он обозначается PF от англ. power factor.

Коэффициент мощности представляет собой безразмерное число в интервале –1 ≤ PF ≤ 1 и часто выражается в процентах. Отрицательный коэффициент мощности указывает, что «нагрузка» в действительности таковой не является (поэтому в кавычках) и реально представляет собой генератор, вырабатывающий электроэнергию, которая отправляется назад в систему. Одним из примеров такой энергии является энергия, получаемая от установленных на крыше жилого дома солнечных батарей. Блок управления солнечными батареями измеряет напряжение, частоту и фазу в сети, синхронизирует свою работу с сетью и выдает в нее лишнюю энергию. В таких случаях современные цифровые электросчетчики показывают отрицательную величину коэффициента мощности.

Если нагрузка чисто резистивная, то напряжение и ток находятся в фазе, коэффициент мощности равен единице и реактивная мощность, которая может быть опережающей или отстающей, равна нулю. Если нагрузка имеет активно-емкостной характер, коэффициент мощности называется опережающим, так как ток опережает напряжение. Если же нагрузка имеет активно-индуктивный характер, то коэффициент мощности называют отстающим, так как ток отстает от напряжения.

Из приведенных выше формул для P и S следует, что для чисто синусоидального напряжения, PF = cos ϕu:

Здесь φu — сдвиг фаз между током и напряжением. Коэффициент мощности уменьшается, если активная мощность уменьшается с увеличением сдвига фаз между напряжением источника питания и током. Коэффициент мощности чисто активной (резистивной) нагрузки равен единице.

Отрицательный сдвиг фаз указывает, что нагрузка емкостная, в которой ток опережает напряжение. Такая нагрузка «отдает» реактивную мощность в систему. Положительный сдвиг фаз показывает, что нагрузка имеет индуктивный характер, ток отстает от напряжения и нагрузка «потребляет» реактивную мощность.

В промышленности коэффициент мощности имеет очень важное значение, так как энергосбытовые компании повышают цены на электроэнергию, если коэффициент мощности падает ниже определенного предела. Работу ведь выполняет активная мощность, а реактивная просто движется туда-сюда между нагрузкой и источником энергии. Образующиеся при этом большие токи повышают потери энергии при передаче. В результате требуется более мощное оборудование для ее получения, а также более толстые провода для передачи, в которых энергия бесполезно нагревает окружающую среду.

Если вам интересно как реальные нелинейные нагрузки искажают форму тока и как описанный выше классический треугольник мощностей превращается в объемную фигуру, откройте наш калькулятор для пересчета вольт-амперов в ватты.

В 50-х и в начале 60-х гг. прошлого века в Европе родители могли подарить на Рождество своему чаду набор для сборки лампового радиоприемника с питанием от сети 220 В…

Не по теме. Когда я писал эту статью, мне попалось мнемоника, которую преподаватели часто используют для облегчения запоминания материала по электротехнике: УЛИЦА (U на L, I на C). Что это за чушь? Зачем вообще бедным студентам зазубривать кто кого опережает? Меня всегда удивляло множество мнемоник, предлагаемых преподавателями студентам для зазубривания вещей, которые студенты должны понимать, а не помнить. На мой взгляд, студенты должны каждый раз думать, когда они отвечают на вопрос, например, о фазовых соотношениях между током и напряжением в емкостной или индуктивной цепи — кто кого опережает: ток опережает напряжение или напряжение опережает ток.

Зазубрить, конечно, проще, да и преподавателю проще проверить зубрежку, чем вникать в тонкости и тому, и другому. Студентам легче, потому что не нужно понимать проблему, достаточно зазубрить простое мнемоническое правило. Преподавателям намного быстрее и, главное, дешевле для самого университета просто проверить ответы на вопросы с несколькими вариантами ответов вместо того, чтобы оценить как студенты поняли материал во время разговора на экзамене.

Не знаю кто как, а я никогда не помнил кто кого опережает и если нужно об этом сказать, то я вспоминаю стрелку мультиметра в режиме измерения сопротивления, которая, если подключить конденсатор достаточно большой емкости, резко отклоняется вправо и потом медленно возвращается назад. Все понятно: ток опережает напряжение — ток уже большой, а напряжение постепенно нарастает. Не нужна мнемоника! Не нужно зубрить электротехнику! Её нужно понимать! Нужно взять аналоговый тестер или цифровой мультиметр с качественным эмулятором стрелочной шкалы, пощупать и всё станет понятно. Можно даже языком пощупать, если напряжение меньше 10 В. Я в детстве щупал и до сих пор живой. Если же студент не хочет брать мультиметр, чтобы понять то, что он изучает, то, как мне кажется, ему лучше вместо электроники изучать историю или иностранные языки. Короче, окончить университет по специальности «умею читать и писать».

Интересно, что в 50-х и в начале 60-х гг. прошлого века в Европе родители могли подарить на Рождество своему чаду набор для сборки радиоприемника на двух лампах с питанием от сети 220 В и никто не боялся, что ребенок получит травму. Может быть потому, что в 50-х и начале 60-х еще были живы воспоминания об ужасной войне и по сравнению с бомбардировками (я хорошо помню мамины рассказы об этом) опасность розетки на 220 вольт не казалась достаточно серьезной? Я в девять лет собрал двухламповый приемник и хорошо помню, что делал это один, без присмотра взрослых. Правда, сам я приемник запустить не смог, так как схемы читать еще не научился и собирал по монтажной схеме, в которой была ошибка. Отец помог его наладить.

Автор статьи: Анатолий Золотков

Как рассчитать потребляемую мощность двигателя

В этой статье мы разберем, что такое мощность трехфазного асинхронного двигателя и как ее рассчитать.

Понятие мощности электродвигателя

Мощность – пожалуй, самый важный параметр при выборе электродвигателя. Традиционно она указывается в киловаттах (кВт), у импортных моделей – в киловаттах и лошадиных силах (л.с., HP, Horse Power). Для справки: 1 л.с. приблизительно равна 0,75 кВт.

На шильдике двигателя указана номинальная полезная (отдаваемая механическая) мощность. Это та мощность, которую двигатель может отдавать механической нагрузке с заявленными параметрами без перегрева. В формулах номинальная механическая мощность обозначается через Р2.

Электрическая (потребляемая) мощность двигателя Р1 всегда больше отдаваемой Р2, поскольку в любом устройстве преобразования энергии существуют потери. Основные потери в электродвигателе – механические, обусловленные трением. Как известно из курса физики, потери в любом устройстве определяются через КПД (ƞ), который всегда менее 100%. В данном случае справедлива формула:

Р2 = Р1 · ƞ

КПД в двигателях зависит от номинальной мощности – у маломощных моделей он может быть менее 0,75, у мощных превышает 0,95. Приведенная формула справедлива для активной потребляемой мощности. Но, поскольку электродвигатель является активно-реактивной нагрузкой, для расчета полной потребляемой мощности S (с учетом реактивной составляющей) нужно учитывать реактивные потери. Реактивная составляющая выражается через коэффициент мощности (cosϕ). С её учетом формула номинальной мощности двигателя выглядит так:

Р2 = Р1 · ƞ = S · ƞ · cosϕ

Мощность и нагрев двигателя

Номинальная мощность обычно указывается для температуры окружающей среды 40°С и ограничена предельной температурой нагрева. Поскольку самым слабым местом в двигателе с точки зрения перегрева является изоляция, мощность ограничивается классом изоляции обмотки статора. Например, для наиболее распространенного класса изоляции F допустимый нагрев составляет 155°С при температуре окружающей среды 40°С.

В документации на электродвигатели приводятся данные, из которых видно, что номинальная мощность двигателя падает при повышении температуры окружающей среды. С другой стороны, при должном охлаждении двигатели могут длительное время работать на мощности выше номинала.

Мы рассмотрели потребляемую и отдаваемую мощности, но следует сказать, что реальная рабочая потребляемая мощность P (мощность на валу двигателя в данный момент) всегда должна быть меньше номинальной:

Р 2 1

Это необходимо для предотвращения перегрева двигателя и наличия запаса по перегрузке. Кратковременные перегрузки допустимы, но они ограничены прежде всего нагревом двигателя. Защиту двигателя по перегрузке также желательно устанавливать не по номинальному току (который прямо пропорционален мощности), а исходя из реального рабочего тока.

Современные производители в основном выпускают двигатели из ряда номиналов: 1,5, 2,2, 5,5, 7,5, 11, 15, 18,5, 22 кВт и т.д.

Расчет мощности двигателя на основе измерений

На практике мощность двигателя можно рассчитать, прежде всего, исходя из рабочего тока. Ток измеряется токовыми клещами в максимальном рабочем режиме, когда рабочая мощность приближается к номинальной. При этом температура корпуса двигателя может превышать 100 °С, в зависимости от класса нагревостойкости изоляции.

Измеренный ток подставляем в формулу для расчета реальной механической мощности на валу:

Р = 1,73 · U · I · cosϕ · ƞ, где

  • U – напряжение питания (380 или 220 В, в зависимости от схемы подключения – «звезда» или «треугольник»),
  • I – измеренный ток,
  • cosϕ и ƞ – коэффициент мощности и КПД, значения которых можно принять равными 0,8 для маломощных двигателей (менее 5,5 кВт) или 0,9 для двигателей мощностью более 15 кВт.

Если нужно найти номинальную мощность двигателя, то полученный результат округляем в бОльшую сторону до ближайшего значения из ряда номиналов.

Р2 > Р

Если необходимо рассчитать потребляемую активную мощность, используем следующую формулу:

Р1 = 1,73 · U · I · ƞ

Именно активную мощность измеряют счетчики электроэнергии. В промышленности для измерения реактивной (и полной мощности S) применяют дополнительное оборудование. При данном способе можно не использовать приведенную формулу, а поступить проще – если двигатель подключен в «звезду», измеренное значение тока умножаем на 2 и получаем приблизительную мощность в кВт.

Расчет мощности при помощи счетчика электроэнергии

Этот способ прост и не требует дополнительных инструментов и знаний. Достаточно подключить двигатель через счетчик (трехфазный узел учета) и узнать разницу показаний за строго определенное время. Например, при работе двигателя в течении часа разница показаний счетчика будет численно равна активной мощности двигателя (Р1). Но чтобы получить номинальную мощность Р2, нужно воспользоваться приведенной выше формулой.

Другие полезные материалы:
Степени защиты IP
Трехфазный двигатель в однофазной сети
Типичные неисправности электродвигателей

Электроэнергия — Электроэнергия — National 5 Physics Revision

Электроэнергия легко перемещается с места на место с помощью проводов или кабелей. Однако электрическая энергия должна быть преобразована в другие формы энергии, прежде чем мы сможем ее использовать.

Скорость передачи (или изменения) энергии называется мощностью — сколько энергии используется в секунду.

\ [Power = \ frac {{Energy}} {{time}} \]

\ [P = \ frac {E} {t} \]

Обозначение мощности : \ (P \) , измеряется в Вт (\ (Вт \)).

Обозначение энергии — \ (E \), оно измеряется в Джоулях (\ (Дж \)).

Символ для времени — \ (t \), он измеряется в секундах (\ (s \)).

Электрическая энергия, передаваемая каждую секунду, определяется умножением напряжения на ток.

\ [Мощность = напряжение \ умножить на ток \]

\ [P = V \ умножить на I \]

Обозначение мощности — \ (P \), оно измеряется в Вт (\ (Вт \)).

Обозначение для напряжения — \ (В \), оно измеряется в Вольтах (\ (В \)).

Обозначение тока — \ (I \), оно измеряется в Ампер (\ (A \)).

Иногда мы не знаем ток или напряжение, поэтому \ (P = IV \) нельзя использовать для расчета мощности. В этих ситуациях мы используем закон Ома, чтобы найти альтернативное уравнение мощности, в котором доступно больше информации. 2}}} {R} \]

Обозначение мощности — \ (P \), оно измеряется в Вт. (\ (W \)).2} R \]

Обозначение для мощности — \ (P \), оно измеряется в Вт (\ (Вт \)).

Обозначение для тока — \ (I \), оно измеряется в Ампер (\ (A \)).

Обозначение сопротивления — \ (R \), оно измеряется в Ом (\ (Ом \)).

Учебное пособие по физике: новый взгляд на электрическую энергию

В предыдущем разделе Урока 3 подробно обсуждалась зависимость тока от разности электрических потенциалов и сопротивления.Ток в электрическом устройстве прямо пропорционален разности электрических потенциалов, приложенной к устройству, и обратно пропорционален сопротивлению устройства. Если это так, то скорость, с которой это устройство преобразует электрическую энергию в другие формы, также зависит от тока, разности электрических потенциалов и сопротивления. В этом разделе Урока 3 мы вернемся к концепции мощности и разработаем новые уравнения, которые выражают мощность через ток, разность электрических потенциалов и сопротивление.

Новые уравнения мощности

В Уроке 2 было введено понятие электроэнергии. Электрическая мощность была определена как скорость, с которой электрическая энергия подается в цепь или потребляется нагрузкой. Уравнение для расчета мощности, подаваемой в цепь или потребляемой нагрузкой, было получено равным

.

P = ΔV • I

(Уравнение 1)

Две величины, от которых зависит мощность, связаны с сопротивлением нагрузки по закону Ома.Разность электрических потенциалов ( ΔV ) и ток ( I ) могут быть выражены в терминах их зависимости от сопротивления, как показано в следующих уравнениях.

ΔV = (I • R) I = ΔV / R

Если выражения для разности электрических потенциалов и тока подставить в уравнение мощности, можно вывести два новых уравнения, которые связывают мощность с током и сопротивлением, а также с разностью электрических потенциалов и сопротивлением.Эти выводы показаны ниже.

Уравнение 2:

P = ΔV • I

P = (I • R) • I

P = I 2 • R

Уравнение 3:

P = ΔV • I

P = ΔV • (ΔV / R)

P = ΔV 2 / R

Теперь у нас есть три уравнения для электрической мощности, два из которых получены из первого с использованием уравнения закона Ома.Эти уравнения часто используются в задачах, связанных с вычислением мощности на основе известных значений разности электрических потенциалов (ΔV), тока (I) и сопротивления (R). Уравнение 2 связывает скорость, с которой электрическое устройство потребляет энергию, с током в устройстве и сопротивлением устройства. Обратите внимание на двойную важность тока в уравнении, обозначенную квадратом тока. Уравнение 2 можно использовать для расчета мощности при условии, что известны сопротивление и ток.Если одно из них неизвестно, то необходимо будет либо использовать одно из двух других уравнений для расчета мощности, либо использовать уравнение закона Ома для расчета количества, необходимого для использования уравнения 2.

Уравнение 3 связывает скорость, с которой электрическое устройство потребляет энергию, с падением напряжения на устройстве и сопротивлением устройства. Обратите внимание на двойную важность падения напряжения, обозначенного квадратом ΔV. Уравнение 3 можно использовать для расчета мощности при условии, что известны сопротивление и падение напряжения.Если одно из них неизвестно, важно либо использовать одно из двух других уравнений для расчета мощности, либо использовать уравнение закона Ома для расчета количества, необходимого для использования уравнения 3.

Концепции на первом месте

Хотя эти три уравнения предоставляют удобные формулы для вычисления неизвестных величин в физических задачах, нужно быть осторожным, чтобы не использовать их неправильно, игнорируя концептуальные принципы, касающиеся схем.Чтобы проиллюстрировать это, предположим, что вам задали такой вопрос: если 60-ваттную лампу в бытовой лампе заменить на 120-ваттную лампу, то во сколько раз ток в цепи этой лампы будет больше? Используя уравнение 2, можно предположить (ошибочно), что удвоение мощности означает, что количество I 2 должно быть удвоено. Таким образом, ток должен увеличиться в 1,41 раза (квадратный корень из 2). Это пример неправильного рассуждения, поскольку он удаляет математическую формулу из контекста электрических цепей.Принципиальное различие между лампочкой на 60 Вт и лампой на 120 Вт заключается не в токе в лампе, а в ее сопротивлении. У этих двух лампочек разные сопротивления; разница в токе — это просто следствие этой разницы в сопротивлении. Если лампы находятся в патроне лампы, который подключен к розетке в США, то можно быть уверенным, что разность электрических потенциалов составляет около 120 вольт. ΔV будет одинаковым для каждой лампы.Лампа мощностью 120 Вт имеет меньшее сопротивление; и, используя закон Ома, можно было бы ожидать, что он также имеет более высокий ток. Фактически, 120-ваттная лампа будет иметь ток 1 А и сопротивление 120 Ом; 60-ваттная лампа будет иметь ток 0,5 А и сопротивление 240 Ом.

Расчеты для 120-ваттной лампы

P = ΔV • I

I = P / ΔV

I = (120 Вт) / (120 В)

I = 1 А

ΔV = I • R

R = ΔV / I

R = (120 В) / (1 А)

R = 120 Ом

Расчеты для 60-ваттной лампы

P = ΔV • I

I = P / ΔV

I = (60 Вт) / (120 В)

I = 0.5 ампер

ΔV = I • R

R = ΔV / I

R = (120 В) / (0,5 А)

R = 240 Ом

Теперь, правильно используя уравнение 2, можно понять, почему удвоение мощности означает, что будет удвоенный ток, поскольку сопротивление также изменяется при замене лампы. Расчет тока ниже дает тот же результат, что и выше.

Расчеты для 120-ваттной лампы

P = I 2 • R

I 2 = P / R

I 2 = (120 Вт) / (120 Ом)

I 2 = 1 Вт / Ом

I = SQRT (1 Вт / Ом)

I = 1 А

Расчеты для 60-ваттной лампы

P = I 2 • R

I 2 = P / R

I 2 = (60 Вт) / (240 Ом)

Я 2 = 0.25 Вт / Ом

I = SQRT (0,25 Вт / Ом)

I = 0,5 А


Проверьте свое понимание


1. Что будет толще (шире) — нить накала 60-ваттной лампочки или 100-ваттная? Объяснять.

2.Вычислите сопротивление и силу тока ночной лампочки мощностью 7,5 Вт, подключенной к розетке в США (120 В).

3. Рассчитайте сопротивление и силу тока электрического фена мощностью 1500 Вт, подключенного к домашней розетке в США (120 В).

4. Коробка на настольной пиле показывает, что сила тока при запуске составляет 15 ампер. Определите сопротивление и мощность двигателя за это время.

5. На наклейке на проигрывателе компакт-дисков написано, что он потребляет ток 288 мА при питании от 9-вольтовой батареи. Какая мощность (в ваттах) у проигрывателя компакт-дисков?

6. Тостер на 541 Вт подключается к бытовой розетке на 120 В. Какое сопротивление (в Ом) тостера?

7.Цветной телевизор имеет ток 1,99 А при подключении к 120-вольтовой электросети. Какое сопротивление (в Ом) у телевизора? А какая мощность (в ваттах) у телевизора?

Как рассчитать электрическую мощность и энергию

Электроэнергия — это скорость выполнения работы. (См. Также: Что такое работа, энергия и мощность?) Электроэнергия — это скорость, с которой электричество работает или дает энергию.В системе СИ единица мощности — ватт, один джоуль в секунду.

Электроэнергия обычно вырабатывается электрическими генераторами, но также может поставляться электрическими батареями. Электроэнергия обычно продается электрическими компаниями в киловатт-часах (3,6 МДж), которые представляют собой произведение мощности в киловаттах на время работы в часах. Электроэнергетические компании измеряют мощность с помощью электросчетчика, который учитывает текущую сумму электроэнергии, доставленной потребителю.

Определение и уравнения для мощности

Электрическая мощность — это скорость выполнения работы, измеряемая в ваттах и ​​обозначаемая буквой P.Термин «мощность» означает «электрическая мощность в ваттах». Электрическая мощность в ваттах, вырабатываемая электрическим током I, состоящим из заряда Q кулонов каждые t секунд, проходящего через разность электрических потенциалов (напряжений) V, составляет:

P = работа, выполненная за единицу времени = VQ / t = (V) (I) или мощность = напряжение x ток или вольт x амперы

где: Q — электрический заряд в кулонах, t — время в секундах, I — электрический ток в амперах, а V — электрический потенциал или напряжение в вольтах

Электроэнергия

Электрическая энергия = мощность x время.Общее количество используемой электроэнергии зависит от общей мощности, используемой всеми вашими электрическими устройствами, и общего времени, в течение которого они используются в вашем доме.

Электрическая энергия измеряется в киловатт-часах

Энергия = Мощность x Время или Киловатт-часы = Киловатт x Часы

Один киловатт-час равен 1000 ватт энергии, используемой в течение одного часа времени.

Как рассчитать стоимость электроэнергии

Из Con Ed Bill — «Мы измеряем вашу электроэнергию по тому, сколько киловатт-часов ((кВтч) вы используете.Один киловатт-час будет освещать 100-ваттную лампочку на 10 часов «.» В 2015 году среднее годовое потребление электроэнергии для потребителя коммунальных услуг в США составило 10812 киловатт-часов (кВтч), в среднем 901 кВтч в месяц . В Луизиане было самое высокое годовое потребление электроэнергии на уровне 15 435 кВтч на бытового потребителя, а на Гавайях было самое низкое — 6 166 кВтч на бытового потребителя ».

ОБРАЗЕЦ ПРОБЛЕМЫ:

Сколько энергии и мощности потребуется для работы кондиционера мощностью 900 Вт в течение 10 часов подряд?

Решение: Энергия = Мощность x Время = 900 Вт x 10 часов = 9000 Вт-часов = 9 кВтч.

КАК ПОНИМАТЬ СЧЕТ НА ЭЛЕКТРОЭНЕРГИЮ

ДАННЫЕ ИЗ СЧЕТА НЕДВИЖИМОСТИ Нью-Йорка 2017 г.

Многое нужно для понимания того, за что вы платите. Это не только стоимость топлива, но и плата за доставку, а также сборы за различные услуги и налоги.

Чтобы объяснить это, мы используем фактический счет Con Ed для небольшой квартиры в Нью-Йорке, использующий Con Edison.

ЗАРЯДЫ НА ЭЛЕКТРОЭНЕРГИЮ

Из Con Edison:

Электроэнергия, которую вы использовали в течение этого 30-дневного расчетного периода с 03 января 2013 г. по 02 февраля 2017 г.

Мы измеряем вашу электроэнергию по тому, сколько киловатт-часов (кВтч) вы используете.

Один кВтч будет светить 100-ваттную лампочку на 10 часов.

02,17 февраля фактическое значение 95175 кВтч

3 января, 17 фактическое значение 94838 кВтч

Таким образом, вы использовали электроэнергию 337 кВтч


ВАШЕ ПОСТАВКА / ПЛАТА — было 337 кВтч при 0,5282 цента / кВтч (это плата за электроэнергию, поставляемую вам Con Ed = 18,83 долларов США

Плата за функцию продавца — плата, связанная с получением кредита на электроэнергию и деятельностью, связанной с взиманием платы, = 1 доллар США.41

GRT и другие налоги = 0,48 доллара США

Общие затраты на поставку = 20,52 долларов США , что составляет 6,1 цента / за кВтч.


ВАША ДОСТАВКА

Базовая плата за обслуживание 16,38 долл. США

Это изменение базовой инфраструктуры системы и услуг, связанных с клиентами, включая учет клиентов, снятие показаний счетчиков и обслуживание счетчиков.

Подача 337 кВтч при 11,0208 ц / кВтч = 37,14 долларов США

Это плата за обслуживание системы, через которую Con ed поставляет вам электроэнергию.

Изменение преимуществ системы при 0,6706 ц / кВтч = 2,26 долл. США

Это возмещает затраты, связанные с деятельностью в области чистой энергии, проводимой штатным сотрудником по исследованиям энергетики штата Нью-Йорк

.

Временная надбавка штата Нью-Йорк 0,1246 цента / кВт · ч = 0,42

Покрывает новые государственные пошлины

GRT и другие доплаты 2,87 долл. США

Итого стоимость доставки $ 69,5


НАЛОГ НА ПРОДАЖУ @ 4,5000%, взимаемый от имени штата Нью-Йорк = 3 доллара США.58

ОБЩАЯ ПЛАТА ЗА ЭЛЕКТРОЭНЕРГИЮ $ 63,15


ОБРАЗЕЦ ВОПРОСА:

Какова стоимость поставки для работы холодильника мощностью 600 Вт в течение 24 часов (при использовании ON) по цене 0,06 цента / кВтч? Примечание. Холодильники не работают постоянно.

Решение: Энергия = Мощность x Время = 600 Вт x 24 часа = 14,4 кВтч x 0,06 цента / кВтч = 864 цента = 8,64 доллара США

Проверьте свое Понимание:

Как оценить потребность в электроэнергии

Когда вы получаете счет за электроэнергию каждый месяц, вы можете не понимать, как именно была рассчитана общая сумма.Каждое устройство в вашем доме вносит свой вклад в общую сумму счета. Чтобы выяснить, какие приборы и устройства потребляют больше всего энергии, вы можете оценить общие требования к мощности , для каждого устройства. Эта оценка также полезна для оценки требований к мощности для альтернативной или резервной системы энергоснабжения.

Оценить потребности в энергии и затраты на питание электронного устройства или прибора очень просто. На задней панели каждого устройства есть этикетка с указанием потребляемой мощности.Это число, которое вам понадобится, чтобы вычислить потребление энергии и требования. Наряду с ваттами вам нужно будет оценить количество часов в день, в течение которых устройство или прибор используется. Если вы предпочитаете не проверять все свои устройства вручную, вы можете приобрести устройство, которое поможет вам оценить потребление энергии. Эти устройства варьируются от простых устройств для измерения мощности до сложных решений для домашнего мониторинга. В этом посте мы предполагаем, что у вас нет доступа к сложному решению для домашнего мониторинга.Если вы новичок в чтении этикеток с энергопотреблением на задней панели ваших приборов и устройств, просмотрите следующие несколько разделов с справочной информацией, чтобы получить представление об основах электротехники и терминологии.

Предпосылки: основы электротехники

Чтобы понять электрические термины, перечисленные на этикетках прибора или устройства, необходимо понять несколько электрических терминов. Основные термины: напряжение, ток и сопротивление:

.

Напряжение (Вольт): Разница потенциальной энергии (заряда) между двумя точками в цепи.Одна точка имеет больше энергии, чем другая, и разница между точками называется напряжением. Напряжение измеряется в вольтах.
Ток (Ампер): Поток электронов (заряд) между двумя точками в цепи. Сила тока измеряется в амперах.
Сопротивление (Ом): Сопротивление — это электрическое сопротивление (сложность) между двумя точками проводника. Сопротивление измеряется в Ом.

Напряжение, ток и сопротивление связаны уравнением, называемым законом Ома:

V = I x R

где V — вольт, I — ток, а R — сопротивление.При описании напряжения, тока и сопротивления часто используется аналогия «вода, текущая в трубе». Ток аналогичен потоку воды, а напряжение — это давление в трубе. Когда напряжение (давление) выше, будет течь более сильный ток. На рисунке 1 показана аналогия с водой с (а) давлением (напряжением) без тока и (б) давлением (напряжением) и током.

Рисунок 1. Механическое изображение напряжения и тока.

Электроэнергия также может быть выражена в единицах мощности, называемых Вт .Ватт — это единица электрической мощности, представленная током в один ампер в цепи с разностью потенциалов в 1 вольт. Мощность связана с напряжением и током следующим уравнением:

P = I x V

где P — мощность, I — ток, а V — вольты. Мощность (электрическая энергия) измеряется в ваттах или киловаттах. Его также можно измерить с течением времени. Например, лампочка мощностью 60 Вт потребляет 60 Вт в определенный момент времени.Киловатт-час (кВтч) — это электрическая энергия, равная мощности, подаваемой одним киловаттом за один час.

Справочная информация: напряжение переменного и постоянного тока

Электрические концепции, которые мы описали до сих пор, являются примерами постоянного тока (DC) . Постоянный ток (DC) — это электрический ток, который течет линейно в постоянном направлении. Существует также другой тип тока, называемый переменным током (AC) , который отличается от постоянного тока, потому что он меняет направление.Рисунок 2 иллюстрирует разницу между этими двумя концепциями. Как показано, постоянное напряжение постоянно. Напряжение переменного тока имеет синусоидальную форму, что означает, что оно изменяется со временем.

Рис. 2. Визуальная разница между постоянным и переменным напряжением.

Мы можем использовать предыдущую аналогию с водой для описания переменного тока; вместо воды, текущей по трубе, вода в трубе перемещается вперед и назад с помощью рукоятки, соединенной с поршнем.На рисунке 3 показана иллюстрация этой концепции. Брызги жидкости могут быть очень быстрыми — 50 или 60 циклов в секунду (50 или 60 Гц). Устройства, которые питаются от топливных элементов или батарей , используют питание постоянного тока; однако устройства, которые подключаются к стене в наших домах, используют переменный ток.

Рисунок 3. Переменный ток Аналогия напряжения (напряжения) и тока.

Проверка необходимой энергии

Чтобы оценить использование энергии в вашем доме, могут помочь следующие источники:

• Счета за электричество
• Рейтинг оборудования
• Ожидаемые профили нагрузки

Посмотрев на свой счет за электроэнергию, вы можете увидеть, как ваши ватты меняются от месяца к месяцу в течение года.Ваше потребление энергии носит сезонный характер и зависит от того, где вы живете. Например, если вы живете в холодном климате, ваши зимние счета могут быть намного выше, чем ваши летние, из-за необходимости в тепле зимой. В жарком климате ваш летний счет может быть намного выше, чем ваш зимний, из-за того, что кондиционер работает все лето.

Каждый прибор или электронное устройство имеет паспортную табличку, на которой указаны напряжение, сила тока, частота и мощность. Обычно они расположены на задней панели устройства.Эти характеристики представляют собой максимальное количество мощности, которое может быть поставлено; следовательно, номинальная мощность, указанная на паспортной табличке, теоретически соответствует 100-процентному использованию. Многие устройства не работают со 100-процентной загрузкой; поэтому использование номинальных значений на паспортной табличке может привести к завышению требований к мощности. Пример паспортной таблички показан на рисунке 4.

Рисунок 4. Паспортные таблички электронного устройства.

Хотя потребление энергии можно рассчитать на основе ваших счетов за электроэнергию и паспортных табличек устройства, фактические измерения дадут более точные данные.Фактические измерения нагрузки можно получить с помощью ватт-часов. Эти фактические измерения нагрузки часто используются для проектирования PV , топливных элементов и систем резервного питания от батарей. Фактическая нагрузка требуется для определения размера и стоимости системы альтернативной энергии . Часто разработчики систем рекомендуют потребителю изменить свои методы энергопотребления, чтобы минимизировать потребление энергии, чтобы фотоэлектрическая система могла быть спроектирована с учетом этих требований вместо установки более крупной системы для компенсации пикового использования.

Расчет потребления энергии

Общее количество энергии, потребляемой вашим домом, можно легко рассчитать, выполнив шесть простых шагов:

1. Укажите количество ватт для каждого прибора или электронного устройства (это называется «нагрузкой» для каждого устройства). Все существующие и планируемые электрические нагрузки должны быть идентифицированы.
2. Оцените среднесуточное использование (количество часов в день, в течение которых прибор или электронное устройство работают)
3. Умножьте мощность устройства на количество часов, в течение которых вы его используете (это даст вам определенное количество «ватт-часов»). Например, если вы используете телевизор на 120 Вт в течение двух часов в день. Вы можете умножить мощность на количество часов, используемых в день, чтобы получить 240 ватт-часов в день.
4. В вашем счете за электричество электричество указано в киловатт-часах. Чтобы сравнить потребление энергии в киловатт-часах, нам нужно будет преобразовать ватт-часы в киловатт-часы. Поскольку 1 киловатт равен 1000 ватт, разделите на 1000, чтобы преобразовать из ватт-часов (Втч) в киловатт-часы (кВтч):

240 Втч / 1000 = 0.24 кВтч

5. Чтобы сравнить эти числа с вашим счетом за электроэнергию, нам нужно преобразовать это число в количество часов, которые прибор или устройство использует в месяц. Например, 0,24 кВтч x 30 дней = 7,2 кВтч в месяц.
6. Чтобы рассчитать затраты на электроэнергию и сравнить их с вашим счетом за электричество, посмотрите на свой счет за электричество, чтобы определить, сколько вы платите за киловатт-час. Если в вашем счете указано, что вы платите 0,12 доллара за киловатт-час, стоимость может быть оценена следующим образом: 7.2 кВтч в месяц x 0,12 доллара США за кВтч = 0,86 доллара США в месяц.

Вы можете организовать эти числа, как в Таблице 1 ниже.

Электрическая нагрузка Мощность (Вт) Среднее ежедневное использование (ч) Среднесуточная энергия (ватт-часы) Среднесуточная энергия (киловатт-часы) Среднемесячная энергия (киловатт-часы) Стоимость в месяц ($)
Телевидение 120 2 240 0.24 7,2 0,86

Таблица 1. Таблица для расчета среднесуточной энергии.

Заполнив Таблицу 1, вы можете получить хорошую оценку количества электроэнергии, которое вы используете каждый месяц, и связанных с этим затрат. Чтобы определить размер системы накопления энергии, вам также нужно будет посмотреть на требуемую пиковую мощность (максимальное количество энергии, которое может потребоваться в день) и продолжительность средней мощности (самый продолжительный период времени, в течение которого средняя мощность нужный).Среднее энергопотребление определяет общее количество энергии, потребляемой за день.

Заключение

В этом посте мы рассмотрели основные электрические термины, такие как напряжение, ток, сопротивление, мощность, постоянный ток (DC) и переменный ток (AC). Затем мы использовали эти концепции для расчета потребности в энергии для прибора или устройства. Эти потребности в энергии можно использовать для оценки общего потребления энергии и связанных с этим затрат на эту энергетическую нагрузку. Расчет этих требований может помочь вам уменьшить ваши счета за электроэнергию и помочь вам определить размер фотоэлектрической, резервной аккумуляторной батареи или другой альтернативной энергетической системы.

Автор Д-р Коллин Шпигель

Доктор Коллин Шпигель — консультант по математическому моделированию и техническому письму (президент SEMSCIO) и профессор, имеющий докторскую степень. и степень магистра инженерных наук. Она имеет семнадцатилетний опыт работы в инженерии, статистике, науке о данных, исследованиях и написании технических статей для многих компаний в качестве консультанта, сотрудника и независимого владельца бизнеса. Она является автором работ « Designing and Building Fuel Cells » (McGraw-Hill, 2007) и «PEM Fuel Cell Modeling and Simulation using MATLAB» (Elsevier Science, 2008).Ранее она владела Clean Fuel Cell Energy, LLC, организацией по топливным элементам, которая обслуживала ученых, инженеров и профессоров по всему миру.

Электрические формулы

Общие электрические единицы, используемые в формулах и уравнениях:

  • Вольт — единица электрического потенциала или движущей силы — потенциал требуется для передачи одного ампера тока через один ом сопротивления
  • Ом — единица сопротивления — один ом — это сопротивление, обеспечиваемое прохождению одного ампера при подаче одного вольт
  • Ампера — единицы тока — один ампер — это ток, который один вольт может передать через сопротивление в один ом
  • Ватт — единица электрической энергии или мощности — один ватт равен произведению одного ампера на один вольт — один ампер тока, протекающего под действием силы одного вольта, дает один ватт энергии
  • Вольт ампер — произведение вольт и ампер как показывают вольтметр и амперметр — в системах постоянного тока вольт-ампер равен ваттам или передаваемой энергии — в системах переменного тока — вольт с и амперы могут быть или не быть на 100% синхронными — при синхронности вольт-амперы равны ваттам на ваттметре — когда несинхронные вольт-амперы превышают ватты — реактивная мощность
  • килоВольт ампер — один киловольт-ампер — кВА — равен 1000 вольт ампер
  • Коэффициент мощности — отношение ватт к вольт-амперам

Электрический потенциал — закон Ома

Закон Ома может быть выражен как:

U = RI (1a) 64

U = P / I (1b)

U = (PR) 1/2 (1c)

Скачать и распечатать Закон Ома

Электрический ток — Закон Ома

I = U / R (2a)

I = P / U (2b)

I = (P / R) 1/2 (2c)

Электрическое сопротивление — закон Ома

R = U / I (3a)

R = U 2 / P 3b)

R = P / I 2 (3c)

Пример — закон Ома

A 12-вольтная батарея обеспечивает питание с сопротивлением 18 Ом .

I = (12 В) / (18 Ом )

= 0,67 (A)

Электроэнергия

P = UI (4a

P = RI 2 (4b)

P = U 2 / R (4c)

где

P = мощность (Вт / Вт, Дж )

U = напряжение (вольт, В)

I = ток (амперы, А)

R = сопротивление (Ом, Ом)

Скачать и распечатать закон Ома

Скачать и распечатать Закон Ома

Электроэнергия

Электроэнергия — это мощность, умноженная на время:

W = P t (5)

whe re

W = энергия (Ws, J)

t = время (с)

Альтернатива — мощность может быть выражена

P = Вт / т (5b)

Мощность потребление энергии потреблением времени.

Пример — потеря энергии в резисторе

Батарея 12 В подключена последовательно с сопротивлением 50 Ом . Мощность, потребляемая резистором, может быть рассчитана как

P = (12 В) 2 / (50 Ом)

= 2,9 Вт

Энергия, рассеиваемая за 60 секунд , может быть рассчитана

W = (2,9 Вт) (60 с)

= 174 Вт, Дж

= 0.174 кВт

= 4,8 10 -5 кВтч

Пример — электрическая плита

Электрическая плита потребляет 5 МДж энергии от источника питания 230 В при включении в течение 60 минут .

Номинальная мощность — энергия в единицу времени — печи может быть рассчитана как

P = (5 МДж) (10 6 Дж / МДж) / ((60 мин) (60 с / мин))

= 1389 Вт

= 1.39 кВт

Ток можно вычислить

I = (1389 Вт) / (230 В)

= 6 ампер

Электродвигатели

КПД электродвигателя

μ = 746 P / P input_w (6)

где

μ = КПД

P л.с. )

или альтернативно

μ = 746 P л.с. / (1.732 VI PF) (6b)

Электродвигатель — мощность

P , 3 фазы = (UI PF 1,732) / 1000 (7)

, где

P 3 фазы = электрическая мощность трехфазного двигателя (кВт)

PF = коэффициент мощности электродвигателя

Электрический двигатель — ток

I , трехфазный = (746 P л.с. ) / (1 .732 В μ PF) (8)

где

I 3-фазный = электрический ток 3-фазного двигателя (амперы)

PF = коэффициент мощности электродвигателя

Как рассчитать трехфазную мощность

Обновлено 12 ноября 2018 г.

Ли Джонсон

Трехфазное питание — широко используемый метод для выработки и передачи электроэнергии, но вычисления, которые вам необходимо выполнить, немногочисленны сложнее, чем для однофазных систем.Тем не менее, при работе с уравнениями трехфазной мощности вам не нужно ничего делать, поэтому вы сможете легко решить любую поставленную вам задачу трехфазного питания. Главное, что вам нужно сделать, это найти ток с учетом мощности в цепи или наоборот.

TL; DR (слишком долго; не читал)

Выполните расчет трехфазной мощности по формуле:

P = √3 × pf × I × V

Где pf — коэффициент мощности, I — ток, В, — напряжение, а P — мощность.

Однофазное и трехфазное питание

Однофазное и трехфазное питание — это термины, описывающие электричество переменного тока (AC). Ток в системах переменного тока постоянно изменяется по амплитуде (т. Е. По размеру) и направлению, и это изменение обычно принимает форму синусоидальной волны. Это означает, что он плавно изменяется с серией пиков и спадов, описываемых синусоидальной функцией. В однофазных системах такая волна всего одна.

Двухфазные системы разделяют его на две части.Каждая секция тока сдвинута по фазе с другой на половину цикла. Поэтому, когда одна из волн, описывающих первую часть переменного тока, находится на пике, другая — на минимальном значении.

Однако двухфазное питание встречается нечасто. Трехфазные системы используют тот же принцип разделения тока на противофазные составляющие, но с тремя вместо двух. Три части тока сдвинуты по фазе на треть цикла каждая. Это создает более сложную схему, чем двухфазное питание, но они одинаково компенсируют друг друга.Каждая часть тока равна по размеру, но противоположна направлению двух других частей, вместе взятых.

Формула трехфазной мощности

Наиболее важные уравнения трехфазной мощности связывают мощность ( P , в ваттах) с током ( I , в амперах) и зависят от напряжения ( В ). В уравнении также присутствует «коэффициент мощности» ( pf ), который учитывает разницу между реальной мощностью (которая выполняет полезную работу) и полной мощностью (которая подается в схему).Большинство типов расчетов трехфазной мощности выполняется с использованием этого уравнения:

P = √3 × pf × I × V

Здесь просто указано, что мощность является квадратным корнем из трех (около 1,732), умноженным на коэффициент мощности (обычно от 0,85 до 1, см. Ресурсы), ток и напряжение. Не позволяйте символам пугать вас, используя это уравнение; Как только вы включите все необходимые составляющие в уравнение, им будет легко пользоваться.

Преобразование кВт в амперы

Допустим, у вас есть напряжение, общая мощность в киловаттах (кВт) и коэффициент мощности, и вы хотите узнать ток (в амперах, А) в цепи.Изменив приведенную выше формулу расчета мощности, мы получим:

I = P / (√3 × pf × V)

Если ваша мощность выражена в киловаттах (т.е. тысячах ватт), лучше либо преобразовать ее в ватт (умножив на 1000) или оставьте его в киловаттах. Убедитесь, что ваше напряжение указано в киловольтах (кВ = вольт ÷ 1000). Например, если у вас коэффициент мощности 0,85, мощность 1,5 кВт и напряжение 230 В, просто укажите мощность как 1500 Вт и вычислите:

I = P / (√3 × pf × V)

= 1500 Вт / √3 × 0.85 × 230 В

Аналогично, мы могли бы работать с кВ (учитывая, что 230 В = 0,23 кВ), и нашли то же самое:

I = P / (√3 × pf × V)

= 1,5 кВт / √3 × 0,85 × 0,23 кВ

Преобразование ампер в кВт

Для обратного процесса используйте форму приведенного выше уравнения:

P = √3 × pf × I × V

Просто умножьте свои известные значения, чтобы найти ответ. Например, при I = 50 A, V = 250 V и pf = 0.9, это дает:

P = √3 × pf × I × V

= √3 × 0,9 × 50 A × 250 В

Поскольку это большое число, преобразуйте его в кВт, используя (значение в Вт) / 1000 = (значение в киловаттах).

19,486 Вт / 1000 = 19,486 кВт

Электроэнергия — веб-формулы

Электрическая мощность определяется по формуле:
P = V · I
Где V — напряжение, а I — ток.

Соответствующие единицы:
ватт (Вт) = вольт (В) · ампер (A)


Мощность также можно определить по следующим формулам:
P = I 2 · R R = P / I 2 I
9055 P
R )
P = V 2 / R R V 901 V = √ ( P · R )


Подробнее об Electric Power 9055 3
Электроэнергия определяется как скорость, с которой работа выполняется источником эл.м.ф. в поддержании тока в электрической цепи. Практическая единица мощности — киловатт и лошадиные силы; где 1 киловатт = 100 ватт и 1 л.с. = 746 ватт.

Если сопротивления (например, электрические приборы) соединены последовательно, ток через каждое сопротивление будет одинаковым. Тогда мощность электрического прибора, P α R и P α V (поскольку V = IR), это означает, что в комбинации сопротивлений серии разность потенциалов и потребляемая мощность будут больше при большем сопротивлении .

Если сопротивления ( i.е. электроприборов) подключены параллельно, разность потенциалов на каждом приборе одинакова. Тогда P α 1 / R и I α 1 / R (как V = IR), что означает, что в параллельной комбинации сопротивлений потребляемый ток и мощность будут больше при меньшем сопротивлении.

Для данного напряжения В, , если сопротивление изменилось с R на ( R / n ), а потребляемая мощность изменилась с P на nP , затем в соответствии с P = V 2 / R , имеем:


P = V 2 / (R / n)) = n (V 2 / R) = nP, где R = R / n и P = nP

Когда приборы питания P 1 , P 2 , P 3 P n включены последовательно с источником напряжения, эффективная потребляемая мощность ( P s ) определяется по формуле:


1/ P s = 1 / P 1 + 1 / P 2 + 1 / P 3 +… + 1 / P n
Для n приборов, каждый из сопротивление R , соединены последовательно с источником напряжения В, рассеиваемая мощность P s тогда задается как:
(1) P s = V 2 / n R

Когда приборы питания

P 1 , P 2 , P 3 P n подключены параллельно источнику напряжения, эффективная мощность израсходовано ( P p ) затем определяется следующим образом:
P s = P 1 + P 2 + P 3 +… + P n 930831 Для приборов n , каждое из которых имеет одинаковое сопротивление R , подключены параллельно к источнику напряжения В , рассеиваемая мощность тогда определяется как:
(2) P p = В 2 / ( R / n) = n V 2 / R

Из (1) и (2) мы имеем P p / P s = n 2 или просто записывается как : P p = n 2 P s .

В соответствии с приведенными выше формулами мы можем объяснить, что:


При группировке ламп серии по заданному источнику напряжения лампа с большей мощностью будет давать меньшую яркость и будет иметь меньший потенциал сопротивления, но тот же ток. , тогда как в параллельном группировке лампочек по данному источнику напряжения лампа большей мощности даст большую яркость и позволит большему току проходить через нее, но будет иметь меньшее сопротивление и такую ​​же разность потенциалов на нем.

Электроэнергия
Электроэнергия определяется как общая выполненная работа или энергия, поставляемая источником ЭДС. при поддержании тока в электрической цепи в течение заданного времени:
Электрическая энергия = электрическая мощность × время = P × t

Таким образом, формула для электрической энергии имеет вид:
Электрическая энергия = P × t = V × I × t = I 2 × R × t = V 2 t / R

S.I единица электрической энергии — джоуль (обозначается J), где 1 джоуль = 1 ватт × 1 секунда = 1 вольт × 1 ампер × 1 секунда
Коммерческая единица электрической энергии — киловатт-час ( кВт · ч, ), где 1 кВтч = 1000 Вт h = 3,6 × 10 6 J = одна единица потребляемой электроэнергии .

Количество единиц потребляемой электроэнергии равно n = (общая мощность × время в часе) / 1000
Стоимость потребления электроэнергии в доме = количество.единиц потребленной электроэнергии × количество на одну единицу электроэнергии.

Теорема о максимальной мощности
В ней говорится, что выходная мощность источника тока максимальна, когда внутреннее сопротивление источника равно внешнему сопротивлению в цепи. Итак, если R — внешнее сопротивление цепи, а r — внутреннее сопротивление источника тока (то есть батареи), то выходная мощность максимальна, когда R = R.

Эта теорема применима ко всем типам источников ЭДС. и связан с выходной мощностью, а НЕ с рассеиваемой мощностью.

Если E — применяемая ЭДС. источника ЭДС. т.е. . батарея с внутренним сопротивлением r и R — внешнее сопротивление, тогда ток в цепи определяется как:
I = E / (R + r)

При максимальной выходной мощности R = r , поэтому имеем:
I = E / (r + r) = E / (2r)
и
максимальная выходная мощность:
P max = I 2 r = E 2 / (4r)

При коротком замыкании аккумулятора мощность равна нулю.В этом случае вся мощность батареи рассеивается внутри батареи из-за ее внутреннего сопротивления. Таким образом, мощность, рассеиваемая внутри батареи, определяется как: P = ( E / r) 2 × r = E 2 / r

КПД источника ЭДС.
КПД источника ЭДС. определяется как отношение выходной мощности (, т. е. , мощность на внешнем сопротивлении цепи, к входной мощности (т. е.мощность, потребляемая от источника ЭДС). Итак,

Где V = падение потенциала на внешнем сопротивлении R,
E = E.M.F. источника тока,
I = ток в цепи.

Если r — внутреннее сопротивление источника ЭДС, тогда
V = IR и E = I (R + r )
или

Когда мощность, полученная от источника, максимальна, тогда R = р. В данной ситуации имеем:

Таким образом максимальный КПД источника эл.м.ф. составляет 50%. Это означает, что для ячейки только половина общей мощности, потребляемой ячейкой, используется для полезных целей, тогда как другая половина рассеивается внутри ячейки.

Пример 1:
Лифт должен поднимать 1000 кг на расстояние 100 м со скоростью 4 м / с. Какую в среднем мощность оказывает лифт во время этой поездки?
Решение:
Работа, проделанная лифтом на 100 метров, легко вычисляется:
W = mgh = (1000) (9.8) (100) = 9,8 × 10 5 Джоулей.

Общее время поездки можно рассчитать по скорости лифта:
t = x / v = 100 м / 4 м / с = 25 с .

Таким образом, средняя мощность определяется по формуле: P = Вт / t = 9,8 × 10 5 / 25s = 3,9 × 10 4 Вт или 39 кВт.

Пример 2:
Считается, что объект в свободном падении достиг конечной скорости , если сопротивление воздуха становится достаточно сильным, чтобы противодействовать всему ускорению свободного падения, в результате чего объект падает с постоянной скоростью.Точное значение конечной скорости зависит от формы объекта, но для многих объектов оно может быть оценено на уровне 100 м / с. Когда объект весом 10 кг достиг предельной скорости, какую силу сопротивление воздуха оказывает на объект?

Решение: Для решения этой проблемы мы будем использовать уравнение P = Fv cos θ , Вместо обычного уравнения мощности, поскольку нам дана скорость объекта. Нам просто нужно вычислить силу, прилагаемую к объекту сопротивлением воздуха, и угол между силой и скоростью объекта.Поскольку объект достиг постоянной скорости, результирующая сила, действующая на него, должна быть равна нулю. Поскольку на объект действуют только две силы: сила тяжести и сопротивление воздуха, сопротивление воздуха должно быть равным по величине и противоположным по направлению силе тяжести. Таким образом, F a = — F G = мг = 98 N, направленным вверх. Таким образом, сила, прилагаемая сопротивлением воздуха, антипараллельна скорости объекта. Таким образом:
P = Fv cos θ = (98) (100) (cos180) = — 9800 Вт

Пример 3: Мощность двигателя насоса составляет 4 кВт.Сколько воды в кг / мин он может поднять на высоту 20 м? (g = 10 м / с 2 )
Решение:
Заданная мощность двигателя P = 4KW = 4000 Вт
Если масса воды, поднятая за одну секунду, = m кг.
Общий объем работы, выполненной при подъеме воды, W = mgh
Мощность P = Вт / т, но t = 1 минута = 60 сек.
4000 = mgh / 60
4000 = (m × 10 × 20) / 60
m = 1200 кг.

Пример 4 : Когда вода течет по трубе, ее скорость изменяется на 5%, найти изменение силы воды?
Решение: Мощность = Сила × Скорость = Скорость изменения количества движения × скорость = {(масса / время) × скорость} x скорость = {(adv) × v} × v = adv 3 где «a» — площадь поперечного сечения, «d» — плотность воды, а «v» — скорость потока воды.
Следовательно, Сила воды прямо пропорциональна кубу скорости воды, поэтому пусть
P = Kv 3 (k — постоянная величина и равна ‘ad’.)
Ведение журнала с обеих сторон
log P = 3log v + log k
Дифференциация с обеих сторон
dP / P = 3dv / v
процентное изменение мощности, dP / P × 100 = 3 × 5% = 15%.

Пример 5 : Кинетическая энергия выбрасываемой воды из плотины используется для вращения турбины. Труба, по которой устремляется вода — 2.4 метра и его скорость 12 м / сек. Предполагая, что вся кинетическая энергия воды используется для вращения турбины, вычислите производимый ток, если эффективность динамо-машины составляет 60% и станция передает мощность 240 кВ. Плотность воды = 10 3 кг / м 3 .
Решение: Учитывая, что
r = радиус трубы = 1,2 м, средняя скорость воды v = 12 м / с
V = 240 кВ = 240 × 10 3 вольт, плотность воды p = 10 3 кг / м 3 .
Теперь кинетическая энергия текущей воды в секунду, т.е.
Power P = (1/2) (массовый расход в секунду) × v 2
= (1/2) pr 2 (l / t) rv 2
= (1/2) pr 2 rv 3
= (1/2) 3,14 × (1,2) 2 × 10 3 × (12) 3 Вт
= 3,9 x 10 6 Вт

Ток в кабелях передачи определяется по формуле:
ток = выходная мощность / напряжение
= (60% мощности P) / (240 × 1000)
= [(60/100) × 3.

Добавить комментарий

Ваш адрес email не будет опубликован. Обязательные поля помечены *